You are on page 1of 26

Krok 2- 2013 (General Medicine) (t.

me/krok2GeneralMedicine) - 1

duodenal ulcer (DU). Recently, the pain character has


Krok 2- 2013 (General Medicine)
changed: it became permanent, persistent, irradiating to
the back. There are general weakness, dizziness, fatigue.
1. A patient with suspected pheochromocytoma has
The patient has put off weight. Objectively: HR- 68/min,
normal blood pressure in the periods between the atacks AP- 120/80 mm Hg. What is most likely cause of
and a tendency to tachycardia. Urine test revealed no deterioration?
pathology. It was decided to use a provocative test with 1. Stenosis development
histamine. What medication should be prepared to provide 2. Exacerbation of duodenal ulcer
emergency care in case of a positive test result?
3. Perforation of duodenal wall
1. Mesatonum 4. Haemorrhage
2. Phentolamine 5. Penetration
3. Nifedipine
4. Prednisolone 6. A 27-year-old patient complains of nasal
5. Pipolphen
haemorrhages, multiple bruises on the anterior surface of
the trunk and extremities, sudden weakness. In blood: Hb-
2. Blood typing resulted in positive isohemagglutination 12
74 g/l, reticulocytes – 16%, RBCs – 2, 5 · 10 /l, platelets
reaction with standard sera of А(II) and В(III) groups and 9
– 30 · 10 /l, ESR- 25 mm/h. What is the most effective
negative reaction with sera of 0(I) and АВ(IV) groups. measure for the treatment of thrombocytopenia?
What is this result indicative of?
1. Vitamin B12
1. The first blood group 2. Hemotransfusion
2. The second blood group 3. Cytostatics
3. The fourth blood group 4. Iron preparations
4. Faulty standard sera 5. Splenectomy
5. The third blood group
7. Several hours before, a 28-year-old patient suddenly
3. During a regular medical examination at a metallurgical developed acute headache and repeated vomiting, then
plant 20% of workers were found overweight (body weight lost consciousness. Objectively: focal neurological
was 5-14% higher than normal), and had early signs of symptoms were not found. Pronounced meningeal
obesity (grade I-II) with Quetelet index from 26 to 30.
symptoms were revealed. AP – 120/80 mm Hg. According
What products share must be reduced in the diet of this to clinical and liquorological findi-ngs the patient was
group of people in the first place in order to normalize their diagnosed with subarachnoid haemorrhage. After
body weight? administration of dehydrants the patient’s condition
1. Meat and fish products somewhat improved. What is the main component of
2. Bakery products
further emergency care?
3. Milk and dairy products 1. Anticoagulants
4. Fruit 2. Antiaggregants
5. Vegetables 3. Coagulants
4. Fibrinolytics
4. A 28-year-old patient complains of infertility. The patient
5. Corticosteroids
has been marri-ed for 4 years, has regular sexual life and
does not use contraceptives but has never got pregnant. 8. A city somatic hospital with 300 beds consists of the
Examination revealed normal state of the genitals, tubal main building which houses the therapeutic and surgical
patency. Basal body temperature recorded over the departments. Several separate buildings house the
course of 3 consecutive menstrual cycles appeared to
maternity, pediatric and radiologic departments that are
have a single phase. What is the most likely cause of connected to the main building by underground walkways
infertility? and above-ground covered skybridges. Specify the
1. Chronic salpingoophoritis building system of the hospital:
2. Genital endometriosis 1. Central-unit
3. Ovulatory menstrual cycle
2. Free
4. Immunological infertility 3. Decentralized
5. Anovulatory menstrual cycle 4. Centralized
5. Combined
5. A 28-year-old patient has been hospitalized for the pain
in the epigastric region. He has a 10-year history of
Krok 2- 2013 (General Medicine) (t.me/krok2GeneralMedicine) - 2

9. A 40-year-old female patient complai-ns of headache, 2. Share of persons subject to preventive examinations
dizziness, muscle weakness, occasional cramps in the 3. Incidence of cardiovascular diseases
extremities. She has been taking anti-hypertensive 4. Share of diseased persons
medications for 10 years. AP- 180/100 mm Hg. Blood 5. Share of workers who actually underwent preventive
potassium – 1,8 millimole/l, sodium – 4,8 millimole/l. In examinations
12
urine: alkaline reaction, the relative densi-ty – 10 , protein
and sugar are not found, WBCs – 3-4 in the field of vision, 13. A 60-year-old patient complains of recurrent pain in
RBCs – 1-2 in the field of vision. Conn’s syndrome is the proximal interphalangeal and wrist joints, their periodic
suspected. Which drug should be chosen for the treatment swelling and reddening that have been observed for 4
of arterial hypertension? years. X-ray picture represents changes in form of
1. Enalapril osteoporosis, joint space narrowing and single usuras.
2. Spironolactone What is the most likely diagnosis?
3. Propanolol 1. Osteoarthritis
4. Hydrochlorothiazide 2. Gout
5. Clonidine 3. Pseudogout
4. Multiple myeloma
10. A 32-year-old patient has developed an acute 5. Rheumatoid arthritis
condition after hypothermia: temperature – 40oC , cough
with 200 ml of sputum per day. The sputum is purulent, 14. An 8-year-old child complains of fever up to 38, 8oC ,
foul-smelling. To the right of the lower lobe the mixed throat pain when swallowing, skin rash. Objectively:
moist rales can be auscultated. Blood tst results: WBCs – lacunar tonsi-llitis, circumscribed hyperaemia and
18, 0 · 10 9/l, ESR – 45 mm/h. Radiographically: in the enanthema of soft palate, pinoint-sized skin rash, mostly
lower lobe of the right lung there is a thick-walled cavity up in the folds and on the flexor surfaces of the extremities,
to 6 cm in diameter with a high horizontal level. What is pale nasolabial triangle. Which antibiotic should be
the most likely diagnosis? administered in the first place?
1. Decomposing lung carcinoma 1. Penicillin
2. Lung abscess 2. Gentamicin
3. Fibro-cavernous pulmonary tuberculosis 3. Ampicillin
4. Infiltrative pulmonary tuberculosis 4. Tetracycline
5. Lung cyst 5. Lincomycin

11. On the 2nd day of illness a 27-year-old patient 15. A newborn’s head is of dolichocephalic shape, that is
complains of unbearable headache, repeated vomiting. front-to-back elongated. Examination of the occipital
Objectively: the patient is in a grave condition. He is region revealed a labour tumour located in the middle
conscious but adynamic. Lies in a forced position with his between the prefontanel and posterior fontanel. Specify
head thrown back. There is no skin rash. Nuchal muscles the type of fetal presentation:
are evidently rigid, there are Kernig’s and Brudzinski’s 1. Face presentation
signs. to– 39, 5oC , Ps- 120/min, AP- 130/80 mm Hg. The 2. Brow presentation
leading syndrome of this disease is caused by: 3. Presentation of the bregma
1. Liquor hypertension 4. Anterior vertex presentation
2. Haemorrhages into the adrenal glands 5. Posterior vertex presentation
3. Affection of the cranial nerve nuclei
4. Liquor hypotension 16. A 24-year-old female patient complains of pain in the
5. Hyperthermy right hypochondrium that is getting worse after taking
meals; nausea, fever up to 37, 7oC , icteric skin, pain in
12. A rural hospital serves 6200 people. Preventive the large joints. These presentations have been observed
examinations were planned for 560 farm workers with for 8 months. Objectively: hepatosplenomegaly. Blood test
different risk factors. 400 workers underwent preventive results: ESR- 47 mm/h, total bilirubin – 86,1 mmol/l, direct
examination. 120 individuals were found to have bilirubin – 42,3 mmol/l. Total protein – 62 g/l, albumins –
cardiovascular problems, 90 of them were registered with 40%, globulins – 60%, gamma globulins – 38%. Viral
a dispensary department for health care maintenance. hepatitis markers were not detected. The antibodies to
Which indicator should be used to assess the organization smooth muscle cells are present. On ultrasound the portal
of health care maintenance at the hospital? vein diameter was of 1 cm. What is the most likely
1. Share of newly diagnosed patient diagnosis?
Krok 2- 2013 (General Medicine) (t.me/krok2GeneralMedicine) - 3

1. Hemachromatosis 5. Extensive
2. Gilbert’s syndrome
3. Cholangiogenic hepatitis 21. A 26-year-old patient with left lower lobe pneumonia
4. Autoimmune hepatitis experiences an acute chest pain on the left during
5. Primary biliary cirrhosis coughing. Objectively: diffuse cyanosis, extension of the
left side of chest. Percussion reveals high tympanitis.
17. A patient operated for acute paraproctitis undergoes Auscultation reveals no respiratory murmurs above the left
antibacterial and detoxification therapy, the local course of side of chest. There is a deviation of the right cardiac
the disease has the positive dynamics. Since the border towards the midclavicular line. What examination
operation the patient has had chills, pyrexia, tachycardia, will be the most informative?
euphoria for five days. The doctor suspected sepsis. What 1. Bronchography
study will confirm the diagnosis? 2. Bronchoscopy
1. X-ray of lungs 3. Pneumotachometry
2. Liver ultrasound 4. Spirography
3. Determining the rate of microbial contamination of 5. X-Ray
wound
4. Blood culture for a pathogen 22. A 28-year-old female patient with a six-year history of
5. Determining the rate of average-weight molecules Raynaud’s syndrome has recently developed pain in the
small joints of hands, difficult movement of food down the
18. A 6-year-old child complains of frequent liquid stool esophagus. What kind of disease can you think of in this
and vomiting. On the 2nd day of desease the child case?
presented with inertness, temperature rise up to 38, 2oC , 1. Pseudotrichiniasis
Ps- 150 bpm, scaphoid abdomen, palpatory painful 2. Systemic lupus erythematosus
sigmoid colon, defecation 10 times a day with liquid, 3. Rheumatoid arthritis
scarce stool with mucus and streaks of green. What is a 4. Systemic scleroderma
provisional diagnosis? 5. Periarteritis nodosa
1. Escherichiosis
2. Shigellosis 23. A patient complains of impaired far vision. Previously
3. Intestinal amebiasis his eyes often turned red and hurt. Objectively: the eyes
4. Salmonellosis are not irritated, the cornea is transparent, anterior
5. Yersiniosis chambers are median deep, their liquid is transparent. The
iris of the right eye has not changed in colour, its pattern is
19. A 25-year-old patient complains of having dull heart unchanged. The pupil is of irregular shape, scalloped.
pain for the last 10 days, dyspnea on mild exertion, Biomicroscopy of the crystalline lens reveals the areas of
palpitations. The diasease developed 2 weeks ago after a opacity and vacuoles. Make a diagnosis:
respiratory infection. Objectively: acrocyanosis, AP- 90/75 1. Diabetic cataract of the right eye
mm Hg, Ps-96/min. Cardiac borders appear to be shifted 2. Radiation cataract of the right eye
to the left and right. Heart sounds are weak and have 3. Senile cataract of the right eye
triple rhythm, there is systolic murmur at the apex. ECG 4. Complicated cataract of the right eye
showed sinus rhythm, complete left bundle branch block. 5. Tetanic cataract of the right eye
What is the most likely diagnosis?
1. Infectious-allergic myocarditis 24. An 8-year-old child with a 3-year history of diabetes
2. Exudative pericarditis was hospitalized in hyperglycemic coma. Specify the initial
3. Infective endocarditis dose of insulin to be administered:
4. Myocarditic cardiosclerosis 1. 0,3-0,4U/kg of body weight per hour
5. Vegetative-vascular dystonia 2. 0,05 U/kg of body weight per hour
3. 0,1-0,2 U/kg of body weight per hour
20. The share of circulatory diseases was 15% in the total 4. 0,4-0,5 U/kg of body weight per hour
of registered diseases among city population. What 5. 0,2-0,3 U/kg of body weight per hour
statistic indicator is it?
1. Intensive 25. A multigravida at 39 weeks of gestation has been
2. Demonstrative delivered to a hospital having a regular labour activity for 8
3. Ratio hours, the waters burst an hour ago. She complains of
4. Average headache, seeing spots. AP is of 180/100 mm Hg. Urine
Krok 2- 2013 (General Medicine) (t.me/krok2GeneralMedicine) - 4

test results: protein – 3,3 g/l, hyaline cylinders. Fetal heart 3. Puncture biopsy of lymph nodes
rate is 140/min, rhythmical. Vaginal examination reveals 4. Mediastinal tomography
complete crevical dilatation, the fetal head is on the pelvic 5. Ultrasound examination of the abdomen
floor, sagittal suture is in line with obstetric conjugate, the
occipital fontanel is under the pubis. What is the optimal 29. A 12-year-old girl undergoes regular
tactics of labour management? gastroenterological check-ups for duodenal ulcer, biliary
1. Vacuum extraction of the fetus dyskinesia. What is the recommended frequency of anti-
2. Conservative labour management relapse treatment?
3. Cesarean section 1. Three times a year
4. Cavity forceps 2. Every 2 months
5. Outlet forceps 3. Twice a year
4. Once a year
26. A patient complains of fatigue, lack of appetite, pain 5. Every 3 months
and burning sensation in the tongue, numbness of the
distal limbs, diarrhea. Objectively: pale skin with lemon- 30. A 40-year-old patient complains of fever up to 39oC ,
yellow tint, face puffiness, brown pigmentation in the form cough with sputum and blood admixtures, dyspnea,
of a “butterfly”, bright red areas on the tongue. The li-ver is weakness, herpetic rash on the lips. Objectively:
3 cm below the costal margin, soft. Blood count: RBCs – respiration rate – 32/min. Under the shoulder blade on the
12 9
1, 5 · 10 /l, colour index – 1,2, WBCs – 3, 8 · 10 /l, right the increased vocal fremitus and dullness of
thrombocytes – 180 · 10 9/l, eosinophils – 0%, stab percussi-on sound were revealed. Auscultation revealed
9
neutrophils – 1%, segmented neutrophils – 58%, bronchial respiration. Blood count: WBCs – 14 · 10 /l,
lymphocytes – 38% monocytes – 3%, RBC macrocytosis. ESR – 35 mm/h. What is the provisional diagnosis?
ESR – 28 mm/h. What diagnosis are these presentation 1. Lung cancer
typical for? 2. Right-sided croupous pneumonia
1. Iron deficiency anemia 3. Cavernous tuberculosis of the right lung
2. B12-deficiency anemia 4. Focal right-sided pneumonia
3. Aplastic anemia 5. Exudative pleuritis
4. Acute erythromyelosis
5. Chronic adrenal failure 31. A 45-year-old patient complains of fever up to 40oC ,
general weakness, headache and spasmodic contraction
27. A 55-year-old male has a 1,5-year history of viral of muscles in the region of a shinwound. The patient got
cirrhosis with symptoms of portal hypertension. Over the injured five days ago when tilling soil and didn’t seek
last month the weakness has progrssed, there appeared medical attention. What kind of wound infection can be
coffee ground vomit. Fibrogastroduodenoscopy revealed suspected?
variceal esophageal haemorrhage. What drug should be 1. Gram-negative
used to reduce the pressure in the portal vein? 2. Tetanus
1. Calcium gluconate 3. Anthrax
2. Reserpine 4. Erysipelas
3. Furosemide 5. Gram-positive
4. Dicynone
5. Vasopressin 32. A 57-year-old patient had an attack of retrosternal pain
that lasted more than 1,5 hours. Objectively: the patient is
28. A 24-year-old patient consulted a doctor about inert, adynamic, has pale skin, cold extremities, poor
enlarged submandibular lymph nodes. Objectively: volume pulse, heart rate – 120/min, AP- 70/40 mm Hg.
submandi-bular, axillary and inguinal lymph nodes are ECG shows ST elevati-on in leads II, III, aVF. What
enlarged. Chest radiograph shows enlarged mediastinal condition are these changes typical for?
12
lymph nodes. In blood: RBCs – 3, 4 · 10 /l, Hb- 100 g/l, 1. Acute pericarditis
colour index – 0,88, thrombocytes – 190 · 109/l, WBCs – 2. Perforated gastric ulcer
9
7, 5 · 10 /l, eosinophils – 8%, stab neutrophiles – 2%, 3. Cardiogenic shock
segmented neutrophiles – 67%, lymphocytes – 23%, ESR 4. Arrhythmogenic shock
– 22 mm/h. What study is required to verify the cause of 5. Acute pancreatitis
lymphadenopathy?
1. Sternal puncture 33.A 49-year-old patient consulted a doctor about difficult
2. Open biopsy of lymph nodes swallowing, voice hoarseness, weight loss. These
Krok 2- 2013 (General Medicine) (t.me/krok2GeneralMedicine) - 5

symptoms have been gradually progressing for the last 3 progressed. The boy had bronchitis at the age of 3. His
months. Objectively: the patient is exhausted, there are father has a history of pulmonary tuberculosis. Objectively:
enlarged supraclavicular lymph nodes. Esophagoscopy body temperature 37, 5oC , conscious, lies supine, with
revealed no oesophageal pathology. Which of the the hip and knee flexed to 90 degrees, nuchal rigidity +6
following studies is most appropriate in this case? cm, partial ptosis of the right eyelid, the dilated right pupil.
1. X-ray of lungs General hyperalgesia is present. Liquor: transparent,
2. Computed tomography of chest, mediastinum pressure – 400 mm of water column, protein – 1,5%,
3. Ultrasound investigation of mediastinum cytosis – 610/3 with predominant lymphocytes, sugar –
4. Multiplanar imaging of esophagus 1,22 mmol/l, chlori-des – 500 mmol/l. What is the most
5. Radioisotope investigation of chest likely diagnosis?
1. Pneumococcal meningitis
34. A 57-year-old patient taken to the surgical department 2. Serous meningitis
by ambulance has been provisionally diagnosed with 3. Epidemic cerebrospinal meningitis
acute intestinal obstruction. Acute pancreatitis is 4. Secondary purulent meningitis
suspected. What is the most informative method of study 5. Tuberculous meningitis
to verify the diagnosis?
1. Plan radiography of stomach 38. A 40-year-old female patient has been hospitalized for
2. Ultrasound attacks of asphyxia, cough with phlegm. She has a 4-year
3. Fibrogastroduodenoscopy history of the disease. The first attack of asphyxia
4. Complete blood count and clinical urinalysis occurred during her stay in the countrysi-de. Further
5. Biochemical blood analysis attacks occurred while cleani-ng the room. After 3 days of
inpatient treatment the patient’s condition has si-gnificantly
35. A 28-year-old patient complains of profuse, painful and improved. What is the most likely etiological factor?
prolonged menstruation. Before and after the menstrual 1. Psychogenic
period there is spotting lasting for 4-6 days. Vaginal 2. Chemicals
examination reveals that the uterus is enlarged 3. Infectious
corresponding to 5-6 weeks of pregnancy, has limited 4. Household allergens
mobility, is painful. Appendages are not palpable. On the 5. Pollen
15th day of the menstrual cycle, the uterus was of normal
size, painless. On account of stated problems and 39. A 58-year-old patient complains of a headache in the
objective examination the patient has been diagnosed with occipital region, nausea, choking, opplotentes. The
internal endometriosis. Which drug should be used for the presentati-ons appeared after a physical exertion.
effective treatment of this patient? Objectively: the patient is excited. Face is hyperemic. Skin
1. Ovidon is pale. Heart sounds are regular, the 2nd aortic sound is
2. Duphaston accentuated. AP- 240/120 mm Hg, HR-92/min.
3. Parlodel Auscultation reveals some fine moist rales in the lower
4. Synoestrolum parts of the lungs. Liver is not enlarged. ECG shows signs
of hypertrophy and left ventricular overload. What is the
36. A 7-year-old female child has developed an acute most likely diagnosis?
condition. She complains of a headache, two onsets of 1. Uncomplicated hypertensic crisis
vomiting. Objectively: deferred reactions, body 2. Acute myocardial infarction, pulmonary edema
temperature – 39, 3oC , pronounced hyperesthesia, 3. Complicated hypertensic crisis, pulmonary edema
nuchal rigidity, positive superior and inferior Brudzinski’s 4. Community-acquired pneumonia
signs, symmetric Kernig’s sign. What is the provisional 5. Bronchial asthma exacerbation
diagnosis?
1. Meningitis 40. While staying in a stuffy room a 19-year-old
2. Food toxicoinfection emotionally labile girl developed severe weakness,
3. Craniocerebral trauma dizziness, blackout, nausea and loss of consciousness
4. Toxic encephalopathy without convulsions. Objectively: the patient is
5. Encephalitis unconscious, the skin is pale, extremities are cold. AP-
90/60 mm Hg, Ps- 96/min, deficient, breathing is shallow.
37. A 7-year-old boy had complained of headache, Pupillary and tendon reflexes are present. There are no
nausea, fatigue for 3 weeks. His condition gradually pathological signs. What is the most likely diagnosis?
deteriorated, headache and general weakness 1. Epileptic attack
Krok 2- 2013 (General Medicine) (t.me/krok2GeneralMedicine) - 6

2. Transient ischemic attack 45. A 48-year-old male patient complains of constant pain
3. Hysterical neurosis in the upper abdomen, mostly on the left, that is getting
4. Vegetovascular paroxysm worse after taking meals; diarrhea, weight loss. The
5. Syncope patient is an alcohol abuser. 2 years ago he had acute
pancreatitis. Blood amylase is 4 g/h·l. Coprogram shows
41. A 48-year-old patient complains of having dull pain in steatorrhea, creatorrhea. Blood glucose is 6,0 mmol/l.
the right lumbar region for over three years. USI shows What treatment is indicated for this patient?
that kidneys are of normal size, at the upper pole of the 1. Insulin
right kidney there is a fluid-containing formation up to 12 2. Gastrozepin
cm in diameter. Excretory urograms show normal 3. No-spa
condition on the left, and the deformation of the superior 4. Contrycal
renal calyces with satisfactory function on the right. What 5. Panzinorm forte
kind of disease can you think of?
1. Multicystic kidney disease 46. A 48-year-old patient complains of weakness,
2. Simple cyst of the right kidney subfebrile temperature, aching pain in the kidney region.
3. Right hydronephrosis These presentations turned up three months ago after
4. Tumour of the right kidney hypothermia. Objectively: kidneys are painful on palpation,
5. Multiple cysts of the right kidney there is bi-laterally positive Pasternatsky’s symptom. Urine
test res: acid reaction, pronounced leukocyturia,
42. A 24-year-old patient had been delivered to the microhematuria, minor proteinuria – 0,165-0,33 g/l. After
thoracic department with a chest injury, a fracture of the the urine sample had been inoculated on conventi-onal
IV, V, VI ribs on the right. Plan radiography shows the fluid media, bacteriuria were not found. What research is most
level in the pleural cavity reaching the III rib on the right. required in this case?
Puncture blood contained clots. What is the optimal 1. Isotope renography
treatment tactics? 2. Daily proteinuria
1. Medical thoracoscopy 3. Urine test for Mycobacterium tuberculosis
2. Pleural puncture 4. Nechiporenko urine test
3. Thoracentesis and thoracostomy 5. Zimnitsky urine test
4. Emergency thoracotomy
5. Hemostatic therapy 47. Examination of a 13-year-old girl revealed acute
glomerulonephritis, nephritic syndrome at the initial stage
43. A 45-year-old female patient has worked as a painter without renal dysfunction. What is the main drug of choice
for 14 years. Contacts with synthetic paint result in face for the basic therapy of this patient?
skin redness, swelling, intense itching, oozing lesions. The 1. Saluretic
symptoms disappear after the exposure to chemical 2. Heparin
agents, but recur even at the smell of paint. The symptom 3. Curantyl
intensity progresses with relapses. Make a provisional 4. Antibiotic
diagnosis: 5. Prednisolone
1. Urticaria
2. Contact-type allergy 48. A puerpera breastfeeding for 1,5 weeks consulted a
3. Toksikodermiya doctor about uniform breast engorgement. Breasts are
4. Simple contact dermatitis painful. The body temperature is of 36, 6oC . Milk
5. Occupational eczema expressing is difficult. What is the most likely diagnosis?
1. Purulent mastitis
44. A man abused alcohol, drank away the property and 2. Lactostasis
wages thus getting himself, his wife and two underage 3. Infiltrative mastitis
children into deep financial problems. He was registered in 4. Fibrocystic mastopathy
a local drug abuse clinic. His wife asked a family doctor, 5. Gangrenous mastitis
what kind of petition she could file in court:
1. On recognition of her husband’s incapacity 49. Six months ago, a 5-year-old child was operated for
2. On recognition of her husband’s partial incapacity CHD. For the last 3 weeks he has complained of fever,
3. On recognition of her husband’s disability heart pain, aching muscles and bones. Examination
4. On recognition of her husband’s incompetence results: “white-coffee”skin colour, auscultation revealed
5. On restriction of her husband’s civil capacity systolic murmur in the region of heart along with a noise in
Krok 2- 2013 (General Medicine) (t.me/krok2GeneralMedicine) - 7

the III-IV intercostal space. Examination of fingertips Which of the following drugs are most effective in this
revealed Janeway lesions. What is your provisional case?
diagnosis? 1. Tranquilizers
1. Infectious endocarditis 2. Antidepressants with a sedative effect
2. Acute rheumatic fever 3. Neuroleptics with an activating effect
3. Sepsis 4. Neuroleptics with a sedative effect
4. Typhoid fever 5. Antidepressants with an activating effect
5. Nonrheumatic carditis
54. The institutions which take part in medical
50. A 35-year-old female patient has gained 20 kg weight examinations can be prevention and treatment facilities,
within a year with the normal diet. She complains of chill, medical board of Ministry of Defense, medical board of
sleepi-ness, shortness of breath. The patient’s mother and Ministry of Home Affairs, medico-social expert
sister are corpulent. Objectively: height – 160 cm, weight – commissions, forensic medical boards etc. What
92 kg, BMI – 35,9. Obesity is uniform, there are no striae. institutions are responsible for temporary disability
The face is amimic. The skin is dry. The tongue is examination?
thickened. Heart sounds are muffled. HR- 56/min, AP- 1. Prevention and treatment facilities
140/100 mm Hg. The patient has constipations, 2. Sanitary-and-prophylactic institutions
amenorrhea for 5 months. TSH-28 mkME/l (normal rate – 3. Medical boards of Ministry of Home Affairs
0,32-5). Crani-ogram shows no pathology. What is the 4. Medico-social expert commissions
etiology of obesity? 5. Medical boards of Ministry of Defense
1. Hypothalamic-pituitary
2. Hypothyroid 55. Doctors of a polyclinic conduct a statistical research of
3. Hypo-ovarian the disease outcomes in two groups of patients (those
4. Hypercorticoid registered with dispensary departments and unregistered
5. Alimentary and constitutive ones), depending on age and level of hygiene. What type
of statistical tables would be most suitable for profound
51. In an urban settlement situated on the riverbank an analysis of the interrelation between the above-mentioned
outbreak of hepatitis A was registered. The disease might variables?
have water origin. This assumption can be confirmed by 1. Developing
growth of the following indicators of water quality: 2. Simple
1. Oxidability 3. Group
2. Escherichia coli index 4. Cross tabulation
3. Presence of benign leptospirosis pathogen 5. Analytical
4. Index of fecal coli-forms
5. Number of coli-phages 56. A 45-year-old female patient complains of frequent
liquid stools with a lot of mucus, pus and blood; pain
52. While lifting a heavy load a 39-year-old patient across the abdomen, loss of 7 kg within 6 months. She
suddenly felt a severe headache, pain in the interscapular has a 1-year history of non-specific ulcerative colitis. What
region, and started vomiting. Objectively: the pulse is group of drugs should be preferred for this patient?
rhythmic, 60/min, AP- 180/100 mm Hg. The patient is 1. Sulfonamides
agitated. He presents wi-th photophobia, hyperacusis. 2. Antibacterial
There are positive Kernig’s and Brudzinski’s signs on both 3. Polyenzymes
9
sides. In blood: WBCs – 10 · 10 /l. CSF is bloody, cytosis 4. Corticosteroids
is 240/3. What is the most likely diagnosis? 5. Nitrofurans
1. Sympathoadrenal crisis
2. Acute hypertonic encephalopathy 57. A 26-year-old patient consulted a doctor abut sore
3. Ischemic stroke throat, fever up to 38, 2oC . A week ago, the patient had
4. Meningococcal meningitis angina, didn’t follow medical recommendations. On
5. Subarachnoid haemorrhage examination, the patient had forced position of his head,
trismus of chewing muscles. Left peritonsillar region is
53. A 26-year-old patient with affective bipolar disorder markedly hyperemic, swollen. What is the provisional
has developed a condition manifested by mood diagnosis?
improvement, behavioural and sexual hyperactivity, 1. Tonsil tumour
verbosity, active body language, reduced need for sleep. 2. Meningitis
Krok 2- 2013 (General Medicine) (t.me/krok2GeneralMedicine) - 8

3. Diphtheria of the pharynx 1. Chronic arterial insufficiency


4. Left-sided peritonsillar abscess 2. Gangrene of the lower extremity
5. Phlegmonous angina 3. Deep vein thrombosis of the lower limbs
4. Acute arterial thrombosis
58. A 64-year-old patient complains of severe pain in the 5. Postthrombophlebitic syndrome
right side of chest, dyspnea, dry cough which appeared
suddenly on exertion. Objectively: the right side of the 62. A 20-year-old patient complains of severe headache,
chest lags behind in the act of breathing. Percussion double vision, weakness, fever, irritability. Objectively:
reveals tympanic sound. Auscultation reveals body temperature is at the rate of 38, 1oC , the patient is
pronouncedly diminished breath sounds on the right. Ps- reluctant to contact, sensitive to stimuli. There is ptosis of
100/min, weak, arrhythmic. AP- 100/50 mm Hg. Cardiac the left eyelid, exotropia, anisocoria S>D, pronounced
sounds are decreased. What disease can be suspected in meningeal syndrome. On lumbar puncture the
this patient? cerebrospinal fluid flowed out under a pressure of 300 mm
1. Right-sided pneumothorax Hg, the fluid is clear, slightly opalescent. 24 hours later
2. Right-sided pleuropneumonia there appeared the fibrinous film. Protein – 1,4 g/l,
3. PATE lymphocytes – 600/3 per mm3, sugar – 0,3 mmol/l. What
4. Right-sided dry pleurisy is the provisional diagnosis?
5. Right-sided hydrothorax 1. Mumps meningitis
2. Lymphocytic Armstrong’s meningitis
59. During the periodic medical examination an assembly 3. Tuberculous meningitis
fitter (works on soldering details) didn’t report any health 4. Meningococcal meningitis
problems. Closer examination revealed signs of asthenic- 5. Syphilitic meningitis
vegetative syndrome. Blood included red blood cells with
basophilic aggregations and a somewhat higher number 63. A baby is 3 months old. The mother consulted a
of reticulocytes, urine had a high concentration of delta- pediatrician about lack of breast milk. After several test
aminolevulinic acid. The complex of symptoms indicates weighings it was found that the child had to receive
the initial stage of chronic intoxication with: supplementary feeding. What is the opti-mal milk formula
1. Tin for this child?
2. Lead 1. Milk formula № 3
3. Mercury 2. Malysh
4. Ethanol 3. Whole cow’s milk
5. Manganese 4. Malutka
5. Milk formula № 2
60. A 10 week pregnant woman was admitted to a hospital
for recurrent pain in the lower abdomen, bloody 64. A 64-year-old patient has been referred to planned
discharges from the genital tracts. The problems turned up hospitalization for general weakness, poor appetite,
after ARVI. The woman was registered for antenatal care. progressive jaundice which appeared over 3 weeks ago
Speculum examination revealed cyanosis of vaginal and wasn’t accompanied by pain syndrome. Objectively:
mucosa, clean cervix, open cervical canal discharging body temperature is at the rate of 36, 8oC , Ps-78/min,
blood and blood clots; the lower pole of the gestational abdomen is soft and painless, the symptoms of peritoneal
sac was visible. What tactics should be chosen? irritation are present, palpation reveals a dramatically
1. Antiviral therapy enlarged, tense gallbladder. What disease are these
2. Pregnancy maintenance therapy symptoms typical for?
3. Curettage of the uterus 1. Chronic cholecystitis
4. Expectant management, surveillance 2. Acute cholecystitis
5. Hysterectomy 3. Cancer of the pancreatic head
4. Duodenal ulcer
61. A 50-year-old patient complains of bursting pain in the 5. Lamblia-induced cholecystitis
left lower limb that is getting worse on exertion, swelling in
the region of shin and foot. Objectively: left shin and foot 65. 4 weeks after myocardial infarction a 56-year-old
are doughy, skin of the lower shin is indurated and has a patient developed acute heart pain, pronounced dyspnea.
bronze tint, subcutaneous veins are dilated, there is an Objectively: the patient’s condition is extremely grave,
ulcer with necrotic masses. What is the most likely there is marked cyanosis of face, swelling and throbbing
diagnosis? of neck veins, peripheral pulse is absent, the carotid artery
Krok 2- 2013 (General Medicine) (t.me/krok2GeneralMedicine) - 9

pulse is rhythmic, 130 bpm, AP is 60/20 mm Hg. is covered with clammy sweat, tremor of the extremities is
Auscultation of heart reveals extremely muffled sounds, present. HR- 110/min, AP- 220/140 mm Hg. Heart sounds
9
percussion reveals heart border extension in both are muffled. Blood test results: WBCs – 9, 8 · 10 /l, ESR –
directions. What is the optimal treatment tactis for this 22 mm/h. Blood glucose – 9,8 millimole/l. What disease is
patient? the most likely cause of this crisis?
1. Puncture of the pleural cavity on the left 1. Preeclampsia
2. Conservative treatment, infusion of adrenomimetics 2. Essential hypertension
3. Pericardiocentesis and immediate thoracotomy 3. Primary hyperaldosteronism
4. Pleural cavity drainage 4. Pheochromocytoma
5. Oxygen inhalation 5. Diabetic glomerulosclerosis

66. On the 2nd day of life a full-term boy developed mild 70. A patient presented to a hospital with a carbuncle of
jaundice of skin and mucous membranes, the general the upper lip. The body temperature is 39oC . There is a
condition of the child is normal. Blood test results: indirect pronounced edema of the upper lip and eyelids. What is
hyperbilirubinemia – 120 mmol/l. The child’s blood group the surgeon’s tactics of choice?
is A(II) Rh(+), his mother’s blood group – B(III) Rh(+). 1. Disclose the carbuncle and administer antibiotics
What is the doctor’s tactics of choice? 2. Administer physiotherapy
1. No drug therapy 3. Hospitalize in the surgical unit
2. Treatment with cholekinetics 4. Disclose the carbuncle and administer out-patient
3. Blood transfusion treatment
4. Treatment with enterosorbents 5. Administer out-patient course of antibiotics
5. Treatment with prednisolone
71. Gastric juice analysis of a 42-year-old male patient
67. A 27-year-old sexually active female complains of revealed absence of free hydrochloric acid at all stages.
numerous vesicles on the right sex lip, itch and burning. Endoscopy revealed pallor, thinning of gastric mucosa,
Eruptions regularly turn up before menstruation and smoothed folds. Microscopically the atrophy of glands with
disappear 8-10 days later. What is the most likely intestinal metaplasia was found. What disease is this
diagnosis? situation typical for?
1. Bartholinitis 1. Menetrier disease
2. Herpes simplex virus 2. Chronic type A gastritis
3. Primary syphilis 3. Chronic type B gastritis
4. Genital condylomata 4. Chronic type C gastritis
5. Cytomegalovirus infection 5. Stomach cancer

68. A 25-year-old patient has been admitted to the 72. In order to reduce weed growth on agricultural land,
hospital with the following problems: weakness, sweating, some herbicides have been used for a long time. In terms
itching, weight loss, enlarged submandibular, cervical, of environmental stability these herbicides are rated as
axillary, inguinal lymph nodes. Objectively: hepatomegaly. stable. Specify the most likely route of their entry into the
Lymph node biopsy revealed giant Berezovsky-Reed- human body:
Sternberg- cells, polymorphocellular granuloma made by 1. Soil-insects-humans
lymphocytes, reticular cells, neutrophils, eosinophils, 2. Soil-animals-humans
fibrous tissue, plasma cells. What is the most likely 3. Soil-protozoa-humans
diagnosis? 4. Soil-microorganisms-humans
1. Lymph node tuberculosis 5. Soil-plants-humans
2. Lymphoreticulosarcoma
3. Cancer metastases to lymph nodes 73. A child is 12 years old. He complains of a dull aching
4. Macofollicular reticulosis pain in the epigastrium and right hypochondrium, that is
5. Lymphogranulomatosis getting worse after taking fatty or fried food, headache,
weakness, nausea, low-grade fever. Abdominal palpation
69. A 32-year-old female complains of dizziness, reveals a marked resistance of muscles in the right
headache, palpitation, tremor. For the last several months hypochondrium, positive Kerr’s, Ortner’s, Murphy’s
she has been under outpatient observation for the symptoms. What is the most likely diagnosis?
increased arterial pressure. Since recently such attacks 1. Acute appendicitis
have become more frequent and severe. Objectively: skin 2. Acute pancreatitis
Krok 2- 2013 (General Medicine) (t.me/krok2GeneralMedicine) - 10

3. Acute gastritis 78. A 59-year-old male complains of heart pain, cough,


4. Viral hepatitis fever up to 38oC . Three weeks ago he suffered a heart
5. Chronic cholecystitis attack. Objectively: Ps- 86/min, rhythmic, blood pressure –
110/70 mm Hg. Auscultation reveals pericardial rub, rales
74. A 42-year-old female patient suffers from micronodular beneath the shoulder blade. Radiography reveals no
9
cryptogenic cirrhosis. Over the last week her condition has pathology. Blood count: WBCs – 10 · 10 /l, ESR – 35
deteriorated: she developed convulsions, mental mm/h. ECG shows no dynamics. It would be most
confusion, progressing jaundice. What study may give reasonable to administer the drugs of the following
reasons for such aggravation? pharmaceutical group:
1. Determination of ALAT and ASAT 1. Antibiotics
2. Determination of serum ammonia 2. Direct anticoagulants
3. Determination of alpha-phetoprotein 3. Glucocorticoids
4. Determination of alkaline phosphatase 4. Fibrinolytics
5. Determination of cholesterol ethers 5. Nitrates and nitrites

75. A 40 week pregnant secundipara is 28 years old. 79. A 38-year-old patient has suddenly developed pain in
Contractions are very active. Retraction ring is at the level the left side of his chest, suffocation. Objectively:
of navel, the uterus is hypertonic, in form of hourglass. On moderately grave condition, Ps- 100/min, AP- 90/60 mm
auscultation the fetal heart sounds are dull, heart rate is Hg, breath sounds on the left cannot be auscultated.
100/min. AP of the parturient woman is 130/80 mm Hg. Chest radiography shows the collapse of the left lung up
What is the most likely diagnosis? to 1/2. What kind of treatment should be administered?
1. Complete hysterorrhexis 1. Pleural puncture
2. Disturbed labour 2. Active thoracostomy
3. Mazolysis 3. Passive thoracostomy
4. Attack of eclampsia 4. Rest, resolution therapy
5. RIisk of hysterorrhexis 5. Operative therapy

76. A patient complains of jerking, throbbing pain in the III 80. A boy was born at 32 weeks gestation. 2 hours after
finger on the right hand. The patient associates these pain birth he developed respiratory distress. The RD severity
onsets with an injury by a nail. The finger skin is assessed by Silverman score was 5. The respiratory
hyperemic and tense, palpation with a bulbous-end probe disorders progressed, respiratory failure couldn’t be
reveals the most painful area. What is the provisional eliminated by Martin-Bouyer CPAP (continuous positive
diagnosis? airway pressure). Ro-gram of lungs shows reticular and
1. Bone felon nodular pattern, air bronhogram. What is the most likely
2. Articular felon cause of respiratory distress syndrome?
3. Pandactylitis 1. Edematous hemorrhagic syndrome
4. Paronychia 2. Hyaline membrane disease
5. Subcutaneous felon 3. Congenital pulmonary emphysema
4. Bronchopulmonary dysplasia
77. A patient complains of being unable to get pregnant 5. Segmental atelectasis
for 5 years. A complete clinical examination gave the
following results: hormonal function is not impaired, 81. An infant is 3 weeks old. Since birth there has been
urogenital infection hasn’t been found, on observed periodical vomiting within a few minutes after
hysterosalpingography both tubes were filled with the feeding. The amount of vomitive masses does not exceed
contrast medium up to the isthmic segment, abdominal that of previous feeding. The infant has age-appropriate
contrast was not visualized. The patient’s husband is body weight. What is the most likely cause of this
healthy. What tactics will be most effective? symptom?
1. ICSI within in-vitro fertilization program 1. Esophageal chalasia
2. Laparoscopic tubal plasty 2. Adrenogenital syndrome
3. Insemination with husband’s sperm 3. Pylorospasm
4. In-vitro fertilization 4. Pyloristenosis
5. Hydrotubation 5. Esophageal achalasia
Krok 2- 2013 (General Medicine) (t.me/krok2GeneralMedicine) - 11

82. Analysis of organization of medical care in a regional 86. A 10-year-old child has been admitted to a hospital
centre has shown that every year about 12% of patients with a closed craniocerebral injury with suspected cerebral
receive inpatient care for diseases that don’t require edema. The patient is in grave condition, unconscious.
round-the-clock monitoring and intensive care. What are The dyspnea, tachycardia, hypertension are present.
the most appropriate organizational changes required to Muscle tone is increased, there is nystagmus, pupillary
address this problem? and oculomotor reactions are disturbed. The mandatory
1. Development of primary care component of intensive care is dehydration. What diuretic
2. Development of medical care forms replacing the in- is adequate in this case?
patient care 1. Hydrochlorthiazide
3. Changes to the statute of outpatient clinics 2. Moduretic
4. Upgrading of hospital facilities 3. Furosemide
5. Restructuring of specialized care 4. Mannitol
5. Spironolactone
83. A 57-year-old female complains of having a sensation
of esophageal compresion, palpitation, difficult breathing 87. A 15-year-old boy feels pain in the region of the left
during eating solid food, occasional vomiting with a full knee joint. Objectively: the soft tissues in the affected
mouth, “wet pillow”sign at night for the last 6 months. region are infiltrated, the joint function is limited.
Objectively: body tempearture – 39oC , height – 168 cm, Radiography reveals a focus of bone destruction in the
weight – 72 kg, Ps- 76/min, АP- 120/80 mm Hg. X-ray distal metaepiphysial segment of the left femur. The
revealed a considerable dilation of esophagus and its destruction is accompanied by periosteal detachment and
constriction in the cardial part. What pathology is most a defect formed within cortex of Codman triangle bone. X-
likely to have caused dysphagia in this patient? ray of chest shows multiple microfocal metastases. What
1. Achalasia cardiae is the most likely pathology?
2. Reflux esophagitis 1. Osteogenic sarcoma
3. Esophageal carcinoma 2. Chondrosarcoma
4. Hiatal hernia 3. Juxtacortical sarcoma
5. Primary esophagism 4. Ewing’s sarcoma
5. Fibrosarcoma
84. 6 people live in a modern flat with the total area of 60
m2. There are TV-video equipment, radios, microwave 88. A 53-year-old female patient complains of cardiac pain
ovens, computer. The residents of the flat complain of bad and rhythm intermissions. She has experienced these
health, occasional headaches, arrhythmia, conjunctivitis. presentations since childhood. The patient’s father had a
What is the most likely cause of this condition? history of cardiac arrhythmias. Objectively: the patient is in
1. Electromagnetic fields grave condition, Ps- 220 bpm, AP- 80/60 mm Hg. ECG
2. Tetrachlor plumbum results: heart rate – 215/min, extension and deformation of
3. Formaldehyde QRS complex accompanied by atrioventricular
4. Carbon dioxide dissociation; positive P wave. Some time later heart rate
5. Anthropotoxins reduced down to 45/min, there was a complete
dissociation of P wave and QRST complex. Which of the
85. A 60 year-old female has been suffering weakness, following will be the most effective treatment?
dizziness, fatigue over the last year. Recently she has 1. Cholinolytics
also developed dyspnea, paresthesia. Objectively: skin 2. Implantation of the artificial pacemaker
and mucous membranes are pale and slightly icteric. The 3. Cardiac glycosides
tongue is smooth due to the loss of lingual papillae. Liver 4. β-adrenoreceptor blocking agents
and spleen are located at the costal margin. Blood count: 5. Calcium antagonists
12
Hb- 70 g/l, RBCs – 1, 7·10 /l, colour index – 1,2,
macrocytes. Administer the patient a pathogenetically 89. A patient is on the sick leave for 4 months
justified drug: continuously from the date of injury. The treatment is
1. Vitamin B1 going to last for 1-2 months more. Who has the right to
2. Ascorbic acid extend the duration of medical certificate for this patient?
3. Vitamin B12 1. Medico-social expert commission
4. Vitamin B6 2. District doctor by agreement with a department chief
5. Iron preparations 3. Medical advisory commission after inpatient treatment
Krok 2- 2013 (General Medicine) (t.me/krok2GeneralMedicine) - 12

4. Medical advisory commission after medico-social patient has been diagnosed with primary uterine inertia.
expert commission examination What is the further tactics of labour management?
5. Medical superintendent 1. Cesarean section
2. Labour stimulation
90. On the first day after a surgery for diffuse toxic goiter a 3. Outlet forceps
patient developed difficulty breathing, cold sweats, 4. Vacuum extraction of the fetus
weakness. Objectively: pale skin, body temperature – 38, 5. Skin-head Ivanov’s forceps
5oC , RR – 25/min, Ps- 110/min, AP-90/60 mm Hg. What
early postoperative complication occurred in the patient? 94. A selective population research study was aimed at
1. Thyrotoxic crisis exploring the effect of air emissions from a metallurgical
2. Compression of the trachea by the hematoma plant on the obstructive bronchitis morbidity in a city. The
3. Hypothyroid crisis calculated correlation coefficient was +0,79. Evaluate the
4. Postoperative tetany strength and direction of the relationship:
5. Acute thyroiditis 1. Inverse, average
2. Direct, average
91. A 77-year-old patient complains of inability to urinate, 3. Inverse, strong
bursting pain above the pubis. The patient developed 4. Direct, strong
acute condition 12 hours ago. Objectively: full urinary
bladder is palpable above the pubis. Rectal prostate is 95. An 18-year-old patient since childhood suffers from
enlarged, dense and elastic, well-defined, with no nodes. bleeding disorder after mi-nor injuries. His younger brother
Interlobular sulcus is distinct. Ultrasonography results: also has bleeding disorders with occasional
prostate volume is 120 cm3, it projects into the bladder haemarthrosis. Which laboratory test will be informative
cavity, has homogeneous parenchyma.Prostate-specific for diagnosis verification?
antigen rate is of 5 ng/ml. What is the most likely disease 1. Fibrinogen rate
that caused acute urinary retention? 2. Determination of prothrombin time
1. Acute prostatitis 3. Clotting time
2. Sclerosis of the prostate 4. Blood clot retraction
3. Prostate carcinoma 5. Thrombocyte count
4. Prostatic hyperplasia
5. Tuberculosis of the prostate 96. A 57-year-old male patient complains of dyspnea on
exertion, heaviness in the right hypochondrium and shin
92. A 58-year-old patient complains of general weakness, edemata towards evening. Objectively: temperature – 38,
loss of 10 kg of weight within 1,5 months, progressive pain 1oC , HR- 20/min, HR=Ps=92/min, AP- 140/90 mm Hg.
in the lumbar region, increased blood pressure up to There is apparent kyphoscoliosis. In the lungs single dry
220/160 mm Hg, subfebrile temperature. Objectively: in rales can be auscultated. Heart sounds are muffled,
the right hypochondrium palpation reveals a formation with rhythmic. ECG: Rv1+Sv5=15 mm. X-ray picture shows the
uneven surface and low mobility; veins of the spermatic bulging of pulmonary artery cone, right ventricle
cord and scrotum are dilated. Blood test results: Hb- 86 enlargement. What is the most likely cause of this
g/l, ESR- 44 mm/h. Urine test results: specific gravity – condition?
1020, protein – 0,99 g/l, RBCs – cover the whole field of 1. Pulmonary heart
vision, WBCs – 4-6 in the field of vision. What is the 2. Mitral stenosis
provisional diagnosis? 3. Dilatation cardiomyopathy
1. Acute glomerulonephritis 4. Atherosclerotic cardiosclerosis
2. Renal tumour 5. Primary pulmonary hypertension
3. Nephroptosis
4. Acute pyelonephritis 97. A 38-year-old male works within the range of ionizing
5. Urolithiasis radiation. At a routi-ne medical examination he presents
12
no problems. In blood: RBCs – 4, 5·10 /l, Hb-80 g/l,
9 9
93. A 30-year-old multigravida has been in labour for 18 WBCs – 2, 8 · 10 /l, thrombocytes – 30 · 10 /l. Decide if
hours. 2 hours ago the pushing stage began. Fetal heart this person can work with sources of ionizing radiation:
rate is clear, rhythmic, 136/min. Vaginal examination 1. The patient is allowed to work with radioactive
reveals the completecervical dilatation, the fetal head in substances for the limited period of time
the pelvic outlet plane. Sagittal suture in line with obstetric 2. Working with radioactive substances and other sources
conjugate, the occipital fontanel is near the pubis. The of ionizing radiation is contraindicated
Krok 2- 2013 (General Medicine) (t.me/krok2GeneralMedicine) - 13

3. The patient can be allowed to work after an extended 102. A 9-month-old child presents with fever, cough,
medical examination dyspnea. The symptoms appeared 5 days ago after a
4. The patient can only work with radioactive substances contact with a person having ARVI. Objectively: the child
of low activity is in grave condition. Temperature of 38oC , cyanosis of
5. The patient is allowed to work with radioactive nasolabial triangle is present. RR- 54/min, nasal flaring
substances while breathing. There was percussion dullness on the
right below the scapula angle, and tympanic sound over
98. A patient is 31 years old. Double-contrast barium the rest of lungs. Auscultation revealed bilateral fine moist
swallow revealed a filling defect on the posterior wall in rales predominating on the right. What is the most likely
the middle segment of esophagus. The defect looked like diagnosis?
a well-defined oval 1,8×1,3 cm large. Mucosal folds 1. Acute laryngotracheitis
adjacent to the defect were intact, peristalsis and elasticity 2. Acute bronchitis
of the walls remained unchanged. Digestive tract 3. ARVI
problems were absent. What is the provisional diagnosis? 4. Acute pneumonia
1. Diverticulum 5. Acute bronchiolitis
2. Esophageal tumour
3. Achalasia cardia 103. During her first visit to the prenatal clinic a pregnant
4. Barrett’s esophagus woman was referred to other doctors for mandatory
5. Esophageal burn consultation. The patient was refered to:
1. Internist, surgeon
99. A 16-year-old girl had had polyuria, polydipsia for 2 2. Dentist, surgeon
months. She had lost 8 kg with a good appetite. The 3. Surgeon, oculist
patient was urgently hospitalized for abdominal pain and 4. Internist, dentist
nausea. Examination revealed glycemia at the rate of 18 5. Therapist, oculist
mmol/l, glycosuria at the rate of 24 g/l. Insulin and infusion
of isotonic solutions of sodi-um chloride and glucose 104. During medical monitoring of labor and professional
eliminated these problems, including thirst. What is the training of 6th grade schoolchildren the school doctor
most likely diagnosis? found that lessons in carpentry workshop are held on
1. Diabetes mellitus type 2 Friday as the fourth and fifth lesson of a total of 90
2. Secondary (symptomatic) diabetes minutes, the motor density at the first lesson is 69%, and
3. Renal glycosuria 65% at the second. Does the lesson organization meet the
4. Diabetes insipidus hygienic requirements?
5. Diabetes mellitus type 1 1. Does not meet the requirements in terms of motor
density at the 1st lesson
100. A 10-year-old patient has a history of mild bronchial 2. Meets the requirements
asthma. During a regular check-up the patient should be 3. Does not meet the requirements as there are two
recommended: consecutive lessons
1. To avoid body tempering procedures 4. Does not meet the requirements in terms of motor
2. To avoid spa treatment density at the 2nd lesson
3. To avoid allergenic food 5. Does not meet the requirements in terms of training
4. Not to go to the seaside duration
5. Not to go in for sports
105. An emergency physician arrived to provide medical
101. Explosion of a tank with benzene at a chemical plant care for a hangman taken out of the loop by his relatives.
has killed and wounded a large number of people. There The doctor revealed no pulse in the carotid arteries,
are over 50 victims with burns, mechanical traumas and absence of consciousness, spontaneous breathing and
intoxication. Specify the main elements of medical care corneal reflexes; cadaver spots on the back and posterior
and evacuation of population in this situation: parts of extremities. A person can be declared dead if the
1. Sorting, evacuation, treatment following sign is present:
2. Isolation, rescue activity, recovery 1. Absence of corneal reflexes
3. Sorting, recovery, rescue activity 2. Pulselessness
4. Medical assistance, evacuation, isolation 3. Cadaver spots
5. Sorting, medical assistance, evacuation 4. Absence of spontaneous breathing
5. Unconsciousness
Krok 2- 2013 (General Medicine) (t.me/krok2GeneralMedicine) - 14

106. A family consists of 5 persons. The husband is a 1. Antibiotic therapy, immobilization and expression of
stope miner. His spouse is a housewife. Their 20-year-old breast milk
daughter works as a kindergarten teacher. Their 18-year- 2. Compress to both mammary glands
old son is a student. The grandmother is a pensioner, she 3. Inhibition of lactation
has diabetes. Which member of this family can be 4. Physiotherapy
primarily classed among the group of persons with a high 5. Opening of the abscess and drainage of the mammary
risk of tuberculosis in the planning of preventive gland
examinations for tuberculosis?
1. Grandmother 111. A 40-year-old female patient complains of having a
2. Wife bulge on the anterior surface of neck for 5 years.
3. Husband Objectively: Ps- 72 bpm, arterial pressure – 110/70 mm
4. Son Hg, in the right lobe of thyroid gland palpation reveals a
5. Daughter mobile 4×2 cm node, the left lobe is not palpable, the
basal metabolic rate is 6%. What is the most likely
107. A 56-year-old female patient complains of recurrent diagnosis?
attacks of intensive pain irradiating along the ureters. 1. Nodular euthyroid goiter
Urine test results: protein – 0,37 g/l, RBCs-20-25 in the 2. Riedel’s thyroiditis
field of vision, WBCs – 12-14 in the field of vision. What 3. Nodular hyperthyroid goiter
method of instrumental diagnostics is the most informative 4. The median cervical cyst
for the diagnosis? 5. Mixed euthyroid goiter
1. Computer tomography
2. USI of kidneys 112. During coal extraction in a mine the concentration of
3. Cystoscopy coal dust in the working area is 450 mg/m3 (MPC is 10
4. Intravenous urography mg/m3). What occupational respiratory disease may
5. Radioisotope renography develop in miners?
1. Byssinosis
108. From urine of a 14-year-old boy with the 2. Allergic nasopharyngitis
exacerbation of secondary obstructive pyelonephritis 3. Anthracosis
Pseudomonas aeruginosa was isolated with a titer of 4. Talcosis
1000000 microbes per 1 ml. Which antibiotic is most 5. Siderosis
advisable to be administered in this case?
1. Ampicillin 113. A 49-year-old patient complains of dyspnea, cough.
2. Cefazolinum There are no sputum discharges. He has repeatedly used
3. Azithromycin salbutamol and intal but with no effect. Objectively: he is
4. Chloramphenicol only able to sit while leaning on the table. Cyanosis of
5. Ciprofloxacin face, acrocyanosis are present. Breathing is shallow,
laboured, in some parts it cannot be auscultated; there are
109. After lifting a load a patient felt undurable pain in the diffuse rales, expiration is significantly prolonged. Heart
loin. He was diagnosed with acute lumbosacral radiculitis. sounds are muffled, tachycardia is present. Ps – 112/min,
Which of the following is contraindicated for this patient? AP- 110/70 mm Hg. Liver is located near the costal arch.
1. Vitamins of B group There are no peripheral edemata. What is your provisional
2. Analgetics diagnosis?
3. Intravenous injection of aminophylline 1. Cardiac asthma
4. Dehydrating drugs 2. Foreign object aspiration
5. Warming procedures 3. Bronchiale asthma, moderate gravity
4. Chronic obstructive bronchitis
110. On the 10th day postpartum a puerperant woman 5. Status asthmaticus
complains of pain and heaviness in the left mammary
gland. Body temperature is 38, 8oC , Ps- 94 bpm. The left 114. A 22-day-old infant had developed red subcutaneous
mammary gland is edematic, the supero-external quadrant nodules from 1,0 to 1,5 cm large on the scalp. Later the
of skin is hyperemic. Fluctuation symptom is absent. The nodules suppurated, body temperature rose up to 37, 7oC
nipples discharge drops of milk when pressed. What is a , there appeared symptoms of intoxication, the regional
doctor’s further tactics? lymph nodes grew bigger. Blood test results: anemia,
Krok 2- 2013 (General Medicine) (t.me/krok2GeneralMedicine) - 15

leukocytosis, neutrophilia, accelerated ESR. What is the irregular menstruation, once every 3-4 months. Over the
most likely diagnosis? last 2 years she presents with hot flashes, sweating, sleep
1. Pemphigus disturbance. Examination revealed no pathological
2. Pseudofurunculosis changes of the internal reproductive organs. Complete
3. Vesiculopustulosis blood count and urinalysis showed no pathological
4. Scalp phlegmon changes. Vaginal smear contained 20-25 leukocytes in the
field of vision, mixed flora. What is the most likely
115. A patient is 60 years old, retired, worked as deputy diagnosis?
director of a research institute. Behavioural changes 1. Cystitis
appeared 2 years ago after the death of her husband: she 2. Bacterial vaginosis
stopped looking after herself and leaving the house; then 3. Menopausal syndrome
she refused to clean the apartment and cook. Mental 4. Vulvitis
status: temporal disorientation. The patient does not 5. Trichomonas colpitis
understand many of the questions, is confused; does not
know how to cook soup or fasten a button. Her speech is 119. Survey radiograph of a 52-year-old worker of an
characterized by stumbling and logoclonia. She does not agglomeration plant (28 years of experience, the
recognize doctors, fellow patients. She cries a lot but can concentration of metal dust is 22-37 mg/m3) shows mildly
not explain the reason for tears. What is the mechanism of pronounced interstitial fibrosis with diffused contrast well-
this pathology? defined small nodular shadows. The patient has no
1. Atrophy of the cerebral cortex complaints. Pulmonary function is not compromised. What
2. Impaired conversion of dopamine to noradrenaline is the provisional diagnosis?
3. Atherosclerotic changes in cerebral vessels 1. Anthraco-silicatosis
4. Disorder of melatonin metabolism 2. Silicatosis
5. Serotonin deficiency 3. Anthracosis
4. Silicosis
116. A 13-year-old boy complains of pain in the upper 5. Siderosis
third of his left thigh, body temperature rise up to 39oC .
There is a swelling in the upper third of thigh and inguinal 120. A 70-year-old patient consulted a doctor about
fold smoothness. The extremity is in a half-bent position. arrhythmic cardiac activity, dyspnea. Objectively: AP-
Active and passive movements are not possible because 150/90 mm Hg, extrasystole arrhythmia (10-12 beats per
of the sharp pain. What is the most likely diagnosis? minute), left ventricular systolic dysfunction (ejection
1. Brodie’s disease fraction at the rate of 42%). Which of antiarrhythmic drugs
2. Intermuscular phlegmon should be administered as initial therapy in this case?
3. Osteosarcoma 1. Flecainide
4. Acute coxitis 2. Encainide
5. Acute hematogenous osteomyelitis 3. Amiodarone
4. Digoxin
117. 5 weeks after hypothermia a 22-year-old patient 5. Moracizine
developed fever, weakness, muscle pain, inability to move
independently. Objectively: tenderness, induration of 121. It is planned to build a multi-disciplinary hospital with
shoulder and shin muscles, restricted acti-ve movements, 500 beds in a town. Specify the location of a polyclinic
erythema on the anterior surface of chest. There is a within the medical centre:
periorbi-tal edema with heliotropic erythema. Gottron’s 1. In the garden and park area
sign is present. What study is required to verify the 2. In the centre of the territory near medical buildings
diagnosis? 3. It is not allowed to place the polyclinic within the centre
1. Aminotransferase activity territory
2. Pneumoarthrography 4. At the main entrance
3. ASO titre 5. In the service zone
4. Rheumatoid factor
5. Muscle biopsy 122. Examination of a Rh-negative pregnant woman at 32
weeks of gestation revealed a four-time rise of Rh-
118. A 49-year-old patient complains of itching, burning in antibody titer within 2 weeks, the titer was 1:64. In the first
the external genitals, frequent urination. The symptoms two pregnancies the patient had experienced antenatal
has been present for the last 7 months. The patient has
Krok 2- 2013 (General Medicine) (t.me/krok2GeneralMedicine) - 16

fetal death due to hemolytic disease. What is the optimal 1. Dejerine-Klumpke palsy
tactics of pregnancy management? 2. Bernard-Horner syndrome
1. Early delivery 3. Duchenne-Erb’s palsy
2. Screening for Rh-antibodies 2 weeks later and early 4. Muscle paresis
delivery in case of further titer rise 5. Total lesion of the brachial plexus
3. Ultrasound for signs of hemolytic disease of the fetus
4. Delivery at 37 weeks of gestation 127. A 28-year-old patient has been taken to a hospital for
5. Introduction of anti-Rh (D) immunoglobulin acute pain in the lower abdomen. There was a brief
syncope. The delay ofmenstruation is 2 months.
123. A 60-year-old male patient, who works as a Objectively: the patient has pale skin, AP- 90/50 mm Hg,
construction worker, complains of pain in the right hip and Ps- 110/min. Lower abdomen is extremely painful. Vaginal
knee joints, that is getting worse on exertion. These examination reveals uterus enlargement. There is positive
presentations have been observed for the last 5 years. Promtov’s sign. Right appendages are enlarged and very
Objectively: the patient is overnourished. Right knee joint painful. Posterior vault hangs over. What is the most likely
is moderately deformed. Examinati-on of other organs and diagnosis?
systems revealed no pathology. Blood tet results: WBCs – 1. Right ovary apoplexy
9
8, 2 · 10 /l, ESR – 15 mm/h. Uric acid – 0,35 mmol/l. What 2. Acute right-sided salpingoophoritis
is the most likely diagnosis? 3. Pelvioperitonitis
1. Reiter’s disease 4. Right-sided tubal pregnancy
2. Rheumatoid arthritis 5. Incipient abortion
3. Gout
4. Deforming osteoarthritis 128. A general practitioner visited a 2-year-old child and
5. Reactive arthritis diagnosed him with measles. The child attends a nursery,
has a 5-year-old sister. What document must be filled in
124. 2 weeks after labour a parturient woman developed for the effective antiepidemic measures in the given health
breast pain being observed for 3 days. Examination locality?
revealed body temperature at the rate of 39oC , chills, 1. House call record (form № 031/o)
weakness, hyperaemia, enlargement, pain and deformity 2. Infant’s record (report form № 112/o)
of the mammary gland. On palpation the infiltrate was 3. Child care sick leave
found to have an area of softening and fluctuation. What is 4. Emergency notification on infectious disease (form №
the most likely diagnosis? 058/o)
1. Serous mastitis 5. Sick child care certificate
2. Lactostasis
3. Mastopathy 129. A 23-year-old female patient has a mental disease
4. Phlegmonous mastitis since the age of 18, the course of disease has no
5. Infiltrative-purulent mastitis remission periods. At a hospital the patient mostly
presents with non-purposeful foolish excitation: she makes
125. A patient underwent stomach resection. During the stereotypic grimaces, exposed, masturbating in front of a
operation, the left upper limb of the patient was abducted loud laugh, repeating the stereotypical abusive shouts.
and fixed to the operating table for anesthetic The patient should be assigned:
management. Postoperatively, the patient developed 1. Neuroleptics
dysfunction of the upper extremity in form of “drop 2. Antidepressants
hand.”This symptom results from the damage of the 3. Tranquilizers
following nerve: 4. Mood stabilizers
1. Median nerve 5. Nootropics
2. Musculocutaneous nerve
3. Ulnar nerve 130. A 10-year-old patient complains of skin itch that
4. Radial nerve occurs at night. Objectively: multiple paired papules,
5. Axillary nerve burrow tracks in the interdigital skin folds, on the
anterolateral surfaces of abdomen and buttocks. Specify
126. A baby born after fast labour has palsy of hand the period of regular medical check-up for pupils of the
muscles. Grasp reflex is absent, as well as hand-to-mouth class where the patient learns:
reflex. Hand sensitivity is absent. What is the most likely 1. 2 weeks
diagnosis? 2. 1 year
Krok 2- 2013 (General Medicine) (t.me/krok2GeneralMedicine) - 17

3. 6 months 135. A 70-year-old patient with a strangulated inguinal


4. 5 days hernia called a therapist in. The strangulation took place
5. 2 months 10 hours ago. There are signs of intestinal obstruction.
The skin over the herniation is hyperemic. What is the
131. A 13-year-old girl complains of fever up to 37, 4oC tactics of choi-ce?
during the last 2 months after recovering from ARVI. 1. Referral to a surgeon
Objectively: malnutrition, diffuse grade II enlargement of 2. Emergency hospitalization to a surgical hospital
the thyroid gland feeling dense on palpation, 3. Reduction of hernia after a narcotic injection
exophthalmos, tachycardia. What kind of pathological 4. Reduction of hernia
syndrome is it? 5. Cold to the hernia, analgesics, antibiotics
1. Hypothyroidism
2. Hypoparathyroidism 136. A 47-year-old patient came to see a doctor on the 7th
3. Thyrotoxicosis day of disease. The disease developed very fast: after the
4. Hyperparathyroidism chill body temperature rose up to 40oC and lasted up to 7
5. Thymomegaly hours, then it dropped abruptly, which caused profuse
sweat. There were three such attacks occuring once in
132. A 45-year-old female patient complaining of general two days. Two days ago the patient arrived from Africa.
weakness, nausea and vomiting hass been delivered to a Objectively: pale skin, subicteric sclera, significantly
hospital by the ambulance. Recently there has been a lack enlarged liver and spleen. What is the cause of fever
of appetite, weight loss. Objectively: hyperpigmentation of attacks in this disease?
skin, blood pressure at the rate of 70/45 mm Hg, 1. Erythrocytic schizogony
bradycardia. Additional studies revealed the reduced 2. Gametocytes
concentration of aldosterone and cortisol in blood, 3. Endotoxin of a causative agent
decreased excretion of 17-ketosteroids and 17- 4. Exotoxin of a causative agent
oxyketosteroids in the urine, hyponatremia, chloropenia, 5. Tissue schizogony
hypokalemia. What therapeutic measures are required?
1. To administer aldosterone 137. In the morning a patient had nausea, abdominal
2. To administer glucocorticoids, mineralocorticoids, and discomfort, single vomiting, dry mouth. In the evening, the
a diet with a high content of cooking salt patient presented with the increasing general weakness,
3. To administer prednisolone double vision, difficult swallowing of solid food.
4. To prescribe a diet with a high content of cooking salt Objectively: ptosis, mydriasis, anisocoria, absence of gag
5. To administer insulin and pharyngeal reflex, dry mucous membranes. The
previous evening the patient had dinner with canned food
133. A 48-year-old patient was found to have diffuse and alcohol. What is the presumptive diagnosis?
enlargement of the thyroid gland, exophthalmia, weight 1. Botulism
loss of 4 kg in 2 months, sweating. Objectively: HR- 2. Food toxicoinfection
105/min, AP- 140/70 mm Hg. Defecation act is normal. 3. Intoxication with unknown poison
What kind of therapy is recommended in this case? 4. Acute ischemic stroke
1. Lugol’s solution 5. Poliomyelitis
2. Mercazolil
3. Thyroxine 138. A patient got flame burns of both hands. On the
4. Propranolol dorsal and palmar surface of hands the blisters are filled
5. Radioiodine with serous fluid. The wrist joint region is hyperemic. The
forearms were not injured. What is the provisional
134. A baby was born by a young smoker. The labour was diagnosis?
complicated by uterine inertia, difficult delivery of the 1. IIb degree flame burn of hands with an area of 2/
baby’s head and shoulders. The baby’s Apgar score was 2. II degree flame burn of hands with an area of 4/
4. Which of the following is a risk factor for a spinal cord 3. III degree flame burn of hands with an area of 4/
injury? 4. IIIa degree flame burn of hands with an area of 4/
1. Uterine inertia 5. II degree flame burn of hands with an area of 2/
2. Pernicious habits
3. Young age of the mother 139. Sanitary examination of the burns unit for adults
4. Chronic hypoxia revealed that 4-bed wards had an area of 28 m2. What is
5. Difficult delivery of the head and shoulders the minimally required ward area for this department?
Krok 2- 2013 (General Medicine) (t.me/krok2GeneralMedicine) - 18

1. 24 m2 144. An employee was on a business trip to another city,


2. 28 m2 where he fell ill and was hospitalized. The sick leave
3. 30 m2 certificate can be issued:
4. 52 m2 1. With the permission of the head doctor of the city
5. 40 m2 hospital
2. By doctor in charge and chief of department
140. An 8-year-old boy has a 2-year history of blotchy 3. With the permission of the deputy chief doctor in
itchy rash appearing after eating citrus fruit. The first charge of medical work
eruption occurred at the age of 6 months after the 4. By doctor in charge
introduction of juices to the baby’s diet. Father has a 5. With the permission of the deputy head doctor after the
history of bronchial asthma, mother – that of allergic disability examination
rhinitis. What is the most likely diagnosis?
1. Quincke’s edema 145. A 30-year-old patient got in a car accident. He is
2. Psoriasis unconscious, pale, has thready pulse. In the middle third
3. Urticaria of the right thigh there is an extensive laceration with
4. Pityriasis Rosea ongoing profuse external arterial bleeding. What urgent
5. Atopic dermatitis actions must be taken to save the life of the patient?
1. Precordial thump
141. A 63-year-old patient with persistent atrial fibrillation 2. Tourniquet above the wound of the right thigh
complains of moderate dyspnea. Objectively: peripheral 3. Artificial lung ventilation
edemata are absent, vesicular respiration is present, heart 4. Plaster bar
rate – 72/min, AP- 140/90 mm Hg. What combination of 5. Tourniquet below the wound of the right thigh
drugs will be most useful in the secondary prevention of
heart failure? 146. A 45-year-old patient, a sailor, was hospitalized on
1. Diuretics, beta-blockers the 2nd day of the disease. A week ago he returned from
2. Cardiac glycosides, ACE inhibitors India. Complains of body temperature of 41oC , severe
3. Cardiac glycosides, diuretics headache, dyspnea, cough with frothy rusty sputum.
4. Beta-blockers, cardiac glycosides Objectively: the patient is pale, mucous membranes are
5. Beta-blockers, ACE inhibitors cyanotic, breathing rate is 24/min, tachycardia is present.
In lungs: dimini-shed breath sounds, moist rales over both
142. A 12-year-old child has been hit on the stomach. The lungs, crepitation. What is the most likely diagnosis?
patient is in moderately grave condition, has a forced 1. Pneumonic plaque
position in bed. The skin is pale, Ps- 122/min. The stress 2. Miliary tuberculosis
on the left costal arch causes a slight pain. There are 3. Influenza
positive Weinert, Kulenkampff symptoms. Macroscopically 4. Ornithosis
the urine is unchanged. What is the most likely diagnosis? 5. Sepsis
1. Spleen rupture, abdominal bleeding
2. Liver rupture, abdominal bleeding 147. A child undergoes in-patient treatment for acute
3. Left kidney rupture, retroperitoneal hematoma staphylococcal destruction of the right lung. Unexpectedly
4. Rupture of the pancreas he develped acute chest pain on the right, dyspnea,
5. Rupture of a hollow organ, peritonitis cyanosis. The right side of chest lags behind in the
respiratory act. Percussion reveals dullness in the lower
143. A 28-year-old male patient complains of parts on the right, bandbox resonance in the upper parts.
regurgitation, cough and heartburn that occurs every day Borders of the relative cardiac dullness are shifted to the
after a meal, when bending forward or lying down. These left. What complication has most likely developed?
problems have been observed for 4 years. Objective 1. Exudative pleuritis
status and laboratory values are normal. FEGDS revealed 2. Spontaneous pneumothorax
endoesophagitis. What is the leading factor in the 3. Pleural empyema
development of this disease? 4. Right-sided pyopneumothorax
1. Hypersecretion of hydrochloric acid 5. Right lung abscess
2. Helicobacter pylori infection
3. Duodeno-gastric reflux 148. A 14-year-old boy with a history of chronic tonsillitis
4. Hypergastrinemia and sinusitis has developed a feeling of heart irregularities
5. Failure of the inferior esophageal sphincter and additional pulse. HR- 83/min. ECG results: regular
Krok 2- 2013 (General Medicine) (t.me/krok2GeneralMedicine) - 19

impulses with no visible P wave that occur every two sinus 1. Right renal pelvis tumour
contractions, QRS complex is dramatically deformed and 2. Bowel volvulus
prolonged to over 0,11 s, T wave is discordant followed by 3. Torsion of the right ovary cyst
a complete compensatory pause. Specify the arrhythmia 4. Acute appendicitis
type: 5. Renal colic
1. Trigeminal extrasystole
2. Partial AV-blockade 153. A 24-year-old female teacher complains of dizziness
3. Complete AV-block and heart pain irradiating to the left nipple. Pain is not
4. Left bundle branch block associated with physical activity and cannot be relieved by
5. Bigeminal extrasystole nitroglycerin, it abates after taking Valocordin and lasts an
hour or more. The patient has a nearly 2-year history of
149. 20 minutes after a normal delivery at 39 weeks a this disease. Objectively: Ps- 76 bpm. AP- 110/70 mm Hg.
puerpera had a single temperature rise up to 38oC . Heart borders are normal, heart sounds are clear. The
Objectively: the uterus is dense, located between the ECG shows respiratory arrhythmia. Radiograph of the
navel and the pubis, painless. Lochia are bloody, of small cervicothoracic spine shows no pathology. Lungs,
amount. Breasts are moderately soft and painless. What is abdomen are unremarkable. What changes in blood
the optimal tactics? formula can be expected?
1. Antibiotic therapy 1. Leukemic hiatus
2. Appointment antipyretic 2. No changes
3. Manual examination of the uterine cavity 3. Thrombocytopenia
4. Further follow-up 4. Increased ESR
5. Expression of breast 5. Leukocytosis

150. A 40-year-old woman with a history of combined 154. Examination of a 38-year-old patient who had been
mitral valve disease with predominant stenosis complains hit with a blunt object on the left side of chest revealed a
of dyspnea, asthma attacks at night, heart problems. At fracture of the X rib with fragments displacement, parietal
present, she is unable to do easy housework. What is the pneumothorax. The patient complains of pain in the left
optimal tactics of the patient treatment? subcostal area. Objectively: the patient is pale, AP- 80/40
1. Antirheumatic therapy mm Hg, Ps- 138/min, of poor volume. USI reveals fluid in
2. Mitral commissurotomy the left abdomen. Splenic rupture is present. What
3. Antiarrhythmia therapy treatment tactics should be chosen?
4. Implantation of an artificial valve 1. Drainage of the left pleural cavity followed by
5. Treatment of heart failure laparotomy
2. Left-sided thoracotomy immediately followed by
151. An 8-year-old girl periodically has sudden short-term laparotomy
heart pain, sensation of chest compression, epigastric 3. Anti-schock actions followed by laparotomy after the
pain, dizziness, vomiting. Objectively: the patient is pale, arterial pressure rise
respiratory rate – 40/min, jugular pulse is present. Ps- 185 4. Immediate laparotomy and alcohol-novocaine block of
bpm, of poor volume. AP- 75/40 mm Hg. ECG taken the X rib
during an attack shows ectopic P waves, QRS wave is not 5. Immediate upper median laparotomy followed by
deformed. At the end of an attack a compensatory pause drainage of the left pleural cavity
is observed. The most likely cause of the attack is:
1. Paroxysmal atrial tachycardia 155. A patient had 4 generalized convulsive seizures
2. Complete AV-block within a day. Between the seizures the patient did not
3. Atrial fibrillation maintain clear consciousness (was in a coma or stupor).
4. Paroxysmal ventricular tachycardia Specify his state:
5. Sinus tachycardia 1. Frequent complex partial seizures
2. Status epilepticus
152. A patient complains about sudden onsets of 3. Frequent generalized seizures
paroxysmal pain in the right lumbar region. 2 hours after 4. Hysterical attacks
the onset the patient had hematuria. Plain radiograph of 5. Frequent jacksonian seizures
the lumbar region shows no pathologi-cal shadows. USI
reveals pyelocaliectasis on the right, the left kidney is 156. A painter working at a motorcar plant has been
normal. What is the most likely diagnosis? diagnosed with moderately severe intoxication with amide
Krok 2- 2013 (General Medicine) (t.me/krok2GeneralMedicine) - 20

compounds of benzene. The in-patient treatment resulted weakness, fever up to 39oC . These mani-festations has
in a considerable health improvement. What expert been present for three days. Objectively: there is an
decision should be made in this case? inflammatory infi-ltrate of the left nasolabial fold 4×4 cm
1. The patient may get back to work providing he will large with a necrotic core in the center, the pronounced
keep to hygiene and sanitary regulations edema of the left side of face, moderate nuchal rigidity.
2. The patient should be issued a sick list for out-patient What treatment is needed?
treatment 1. Antibiotics, surgery under general anesthesia
3. The patient should be referred to the medio-social 2. Wet-to-dry dressings with antiseptics, aspirin
expert commission for evaluation of percentage of work 3. Physiotherapy, immunomodulators
capicty loss 4. Hospitalization, antibiotics, anti-coagulants, rest
4. The patient should be referred to the medio-social 5. Antibiotics, angioprotectors, daily dressings
expert commission for attributing the disability group
because of an occupational disease 161. Against the background of angina a patient has
developed pain in tubular bones. Examination revealed
157. A 37-year-old patient complains of acute pain in the generalized enlargement of lymph nodes, hepatolienal
12
region of genitals, swelling of the labia, pain when walking. syndrome, sternalgia. In blood: RBCs – 3, 6 · 10 /l, Hb-
9 9
Objectively: body temperature is 38, 7°C , Ps- 98/min. In 87 g/l, thrombocytes 45 10 /l, WBCs – 13 · 10 /l, blasts –
the interior of the right labia there is a dense, painful 87%, stab neutrophils – 1%, segmented neutrophils – 7%,
tumour-like formation 5,0×4,5 cm large, the skin and lymphocytes – 5%, ESR 55 mm/h. What is the most likely
mucous membrane of genitals is hyperemic, there are di-agnosis?
profuse foul-smelling discharges. What is the most likely 1. Erythremia
diagnosis? 2. Acute leukemia
1. Labial furuncle 3. Chronic myeloid leukemia
2. Acute bartholinitis 4. Multiple myeloma
3. Carcinoma of vulva 5. Chronic lymphocytic leukemia
4. Acute vulvovaginitis
5. Bartholin gland cyst 162. A young woman with seborrhea adiposa has
numerous non-itchy light brown and white spots with clear
158. A 58-year-old female patient came to the antenatal outlines and defurfuration on the torso and shoulder skin.
clinic complaining of bloody light-red discharges from the What is the provisional diagnosis?
genital tracts. Menopause is 12 years. Gynaecological 1. Seborrheic dermatitis
examination revealed age involution of externalia and 2. Tinea corporis
vagina; uterine cervix was unchanged, there were scant 3. Vitiligo
bloody discharges from uterine cervix, uterus was of 4. Pityriasis versicolor (scaly skin disease)
normal size; uterine appendages were not palpable; 5. Pityriasis rosea
parametria were free. What is the most likely diagnosis?
1. Atrophic colpitis 163. An emergency doctor has diagnosed a 32-year-old
2. Granulosa cell tumor of ovary woman with generalized convulsive status epilepticus. The
3. Uterine carcinoma deterioration in the patient’s condition is caused by a
4. Cervical carcinoma sudden gap in the epilepsy treatment. Specify the doctor’s
5. Abnormalities of menstrual cycle of climacteric nature further tactics:
1. Outpatient monitoring by a neuropathologist
159. A 24-year-old patient got a puncture injury below the 2. Outpatient monitoring by a neurosurgeon
Poupart’s ligament accompanied by intense arterial 3. Hospitalization in the department of neurosurgery
bleeding. The best method to temporarily stop the 4. Hospitalization in the intensive care unit
bleeding in the patient would be: 5. Hospitalization in the department of neurology
1. Esmarch’s tourniquet
2. Compression band 164. Examination of a newborn revealed skin redness that
3. Wound suturing appeared immediately after birth and reached the
4. Maximum limb bending maximum intensity on the second day of life. What is your
5. Compressing a blood vessel with a clamp provisional diagnosis?
1. Annular erythema
160. A 28-year-old patient consulted a surgeon about 2. Transient erythema
pain, edema and hyperemia of the left side of his face, 3. Toxic erythema
Krok 2- 2013 (General Medicine) (t.me/krok2GeneralMedicine) - 21

4. Simple erythema extirpation of the uterus with its appendages. What is the
5. Erythema nodosum most likely diagnosis?
1. Early pathological menopause
165. A 40-year-old patient is registered in a narcological 2. Secondary psychogenic amenorrhea
dispensary. Somatically: skin is dramatically hyperemic, 3. Premenstrual syndrome
sclera are injected, hyperhidrosis is present. AP-140/100 4. Physiological premenopause
mm Hg, heart rate – 100/min. Mental state: autopsychic 5. Post-castration syndrome
orientation is intact, allopsychic orientation is distorted.
The patient presents with motor anxiety. There is a look of 169. Factory’s sectorial doctor chooses a group of
fear on his face. He refuses to talk about his problems and chronically ill people. He takes into account the duration of
asks to release him immediately, because he “may be etiologically related cases with a temporary disability over
killed.”This state developed a day after a regular drinking the last year in each of the workers. The employees will
bout. What is your provisional diagnosis? fall into this group if the duration is:
1. Organic delirium 1. 10 days or more
2. Alcoholic paranoid 2. 30 days or more
3. Paranoia 3. 20 days or more
4. Delirium tremens 4. 60 days or more
5. Alcoholic hallucinosis 5. 40 days or more

166. X-ray picture of chest shows a density and an abrupt 170. A 3-month-old girl presents with rhinitis, dyspnea, dry
decrease in the upper lobe of the right lung. The middle cough. These manifestations has been observed for two
and lower lobe of the right lung exhibit significant days. Objectively: the child has pale skin, acrocyanosis,
pneumatization. The right pulmonary hilum comes up to shallow respiration at the rate of 80/min. Percussi-on
the dense lobe. In the upper and middle parts of the left reveals handbox resonance over the whole surface of
pulmonary field there are multiple focal shadows. In the lungs, massive fine rales. What is the most likely
basal region of the left pulmonary field there are clear diagnosis?
outlines of two annular shadows with quite thick and 1. Foreign body of the airway
irregular walls. What disease is this X-ray pattern typical 2. Mucoviscidosis
for? 3. Acute bronchiolitis
4. Pneumonia
1. Abscessing pneumonia 5. Acute bronchitis
2. Atelectasis of the right upper lobe
3. Fibro-cavernous pulmonary tuberculosis 171. A patient complains of frequent, bulky, frothy stools
4. Peripheral cancer with greenish mucus, cramping pain in the umbilical
5. Pancoast tumour region, abdominal murmur, body temperature at the rate
of 39oC . The patient associates the disease with
167. Examination of a 43-year-old man objectively consumption of soft-boiled eggs. What is the most likely
revealed pallor of skin and mucous membranes, loss of pathogen?
tongue papillae, transverse striation of fingernails, cracks 1. Enteropathogenic E.Coli
in the mouth corners, tachycardia. Blood test results: Hb- 2. Salmonella
90 g/l, anisocytosis, poikilocytosis. The most likely 3. Yersinia
causative agent of this state is inadequate intake of: 4. Shigella
1. Selene 5. Vibrio cholerae El Tor
2. Iron
3. Magnesium 172. A 13-year-old girl complains of periodic prickly pain in
4. Zinc the heart region. Percussion revealed no changes of
5. Copper cardiac borders. Auscultation revealed arrhythmic
enhanced heart sounds, extrasystole at the 20-25 cardiac
168. A 38-year-old female patient complains about hot impulse. ECG showed the sinus rhythm, impaired
flashes and feeling of intense heat arising up to 5 times a repolarization, single supraventricular extrasystoles at
day, headaches in the occipital region along with high rest. What is the most likely diagnosis?
blood pressure, palpitations, dizziness, fatigue, irritability, 1. Myocardial degeneration
memory impairment. 6 months ago the patient underwent 2. Intoxication syndrome
3. Vegetative-vascular dysfunction
Krok 2- 2013 (General Medicine) (t.me/krok2GeneralMedicine) - 22

4. Nonrheumatic carditis 4. Sternal puncture


5. Rheumatism 5. Immunogram

173. A week ago a 65-year-old patient suffered an acute 177. Full-term pregnancy. Body weight of the pregnant
myocardial infarction, his general condition deteriorated: woman is 62 kg. The fetus has the longitudinal position,
he complains of dyspnea at rest, pronounced weakness. the fetal head is pressed against the pelvic inlet.
Objectively: edema of the lower extremities, ascites is Abdominal circumference is 100 cm. Fundal height is 35
present. Heart borders are extended, paradoxical pulse is cm. What is the approximate weight of the fetus?
2 cm displaced from the apex beat to the left. What is the 1. 4 kg
most likely diagnosis? 2. 2 kg 500 g
1. Acute cardiac aneurysm 3. 3 kg
2. Cardiosclerotic aneurysm 4. 3 kg 500 g
3. Acute pericarditis 5. 4 kg 500 g
4. Recurrent myocardial infarction
5. Pulmonary embolism 178. A 62-year-old male patient complains of intense pain
in the left leg that suddenly arose three hours before, leg
174. 10 days after birth a newborn developed a sudden numbness and coldness. During the year there has been
fever up to 38, 1oC . Objectively: the skin in the region of pain in the leg while walking, hypersensitivity to cooling.
navel, abdomen and chest is erythematous; there are Objectively: the left foot and shin have marbled skin,
multiple pea-sized blisters with no infiltration at the base; subcutaneous veins are collapsed. The foot is cold, active
single bright red moist erosions with epidermal fragments movements of the foot and toes are preserved. Pulse is
on the periphery. What is your provisional diagnosis? present only on the femoral artery. There is rough systolic
1. Syphilitic pemphigus murmur above the artery. Make a provisional diagnosis:
2. Atopic dermatitis 1. Acute thrombophlebitis
3. Epidemic pemphigus of newborn 2. Acute occlusion of the left femoral artery
4. Streptococcal impetigo 3. Stenosis of the left popliteal artery
5. Vulgar impetigo 4. Acute arterial thrombosis ileofemoralny
5. Occlusive disease
175. After myocardial infarction, a 50-year-old patient had
an attack of asthma. Objectively: bubbling breathing with 179. Hygienic expertise of a sample taken from the batch
frequency of 32/min, cough with a lot of pink frothy of grain revealed that 2% of grains were infected with
sputum, acrocyanosis, swelling of the neck veins. Ps- microscopic Fusarium fungi. On the ground of laboratory
108/min, AP- 150/100 mm Hg. Heart sounds are muffled. analyses this batch of grain should be:
Mixed moist rales can be auscultated above the entire 1. Tested for toxicity
lung surface. What drug would be most effective in this 2. Used for forage production
situation? 3. Used for ethanol production
1. Nitroglycerin intravenously 4. Sold without restrictions
2. Pentamin intravenously 5. Destroyed
3. Strophanthin intravenously
4. Dopamine intravenously 180. A newborn (mother’s I pregnancy) weighing 3500 g
5. Aminophylline intravenously presents with jaundice, lethargy, reduced reflexes.
Objectively: second grade jaundice of skin with saffron
176. Medical examination of a 19-year-old worker tint, liver – +2cm, spleen – +1 cm. Urine and feces are
12
revealed generalized lymphadenopathy mainly affecting yellow. Blood count: Hb- 100 g/l, RBCs – 3, 2 · 10 /l,
9
the posterior cervical, axillary and ulnar lymph nodes. WBCs – 18, 7 · 10 /l, mother’s blood type – 0(I) Rh(+),
There are multiple injection marks on the elbow bend skin. baby’s blood type – А(II) Rh(-), bilirubin – 170 mmol/l,
The man denies taking drugs, the presence of injection indirect fraction. ALT, AST rates are normal. What disease
marks ascribes to influenza treatment. Blood count: is the child most likely to have?
12 9
RBCs- 3, 2· 10 /l, Hb- 100 g/l, WBCs- 3, 1 · 10 /l, 1. Perinatal hepatitis
moderate lymphopenia. What study is required in the first 2. Hemolytic disease of newborn, Rh-conflict
place? 3. Biliary atresia
1. ELISA for HIV 4. Hemolytic disease of newborn, AB0-conflict
2. Lymph node biopsy 5. Physiologic jaundice
3. X-ray of lungs
Krok 2- 2013 (General Medicine) (t.me/krok2GeneralMedicine) - 23

181. A 13-year-old girl has a 5-year histrory of pain in the 1. Ciprofloxacin


right hypochondrium irradiating to the right shoulder blade. 2. Tetracycline
The pain attacks are usually associated with diet 3. Erythromycin
violations, they are short and can be easily relieved by 4. Oxacillin
antispasmodic drugs. During a pain attack, palpation of 5. Tseporin
the abdomen is painful, the pain is most intensive in the
projection of the gallbladder. What is the most likely 186. A 12-year-old boy has a 6-year history of insulin-
diagnosis? dependent diabetes. The disease is labile. Since recently
1. Chronic gastroduodenitis there have been periodical rises in blood pressure.
2. Chronic pancreatitis Microalbuminuria test gave positive results. The patient’s
3. Chronic cholecystitis condition corresponds with the following stage of diabetic
4. Biliary dyskinesia nephropathy:
5. Duodenal ulcer 1. Stage III - early-stage nephropathy
2. V stage - chronic renal failure
182. Chief physician of a polyclinic encharged a district 3. Stage I - renal hypertrophy and hyperfunction
doctor with a task to determine the pathological 4. Stage IV - advanced clinical nephropathy
prevalence of disease N in his district. What document 5. Stage II - histological changes in the kidneys
allows to estimate the disease prevalence in the
population of a medical district? 187. A 45-year-old patient with acute abscess of the left
1. Statistic coupons (+) and (-) lung has suddenly developed acute chest pain and
2. Statistic coupons (-) dyspnea while coughing, tachycardia has increased. The
3. Vouchers for medical appointments control Ro-gram shows collapsed left lung, the air in the
4. Statistic coupons (+) left pleural cavity and a horizontal fluid level. What is the
5. Prophylactic examinations register mechanism of this complication?
1. Atelectasis of the left lung
183. A 22-year-old vegetarian patient with signs of 2. Inflammation spread to the visceral pleura
malnutrition consulted a doctor about smell and taste 3. Acute cardiovascular insufficiency
distortion, angular stomatitis. Objectively: expressively 4. Abscess burst into the pleural cavity
blue sclerae. The patient was diagnosed with iron 5. Bullae rupture of the left lung
deficiency anemia. What is the dominating clinical
syndrome? 188. A patient with fibromyoma of uterus sized up to 8-9
1. Sideropenic weeks of pregnancy consulted a gynaecologist about
2. Anaemic acute pain in the lower abdomen. Examination revealed
3. Haemologic pronounced positive symptoms of peritoneal irritation, high
4. Haemolytic leukocytosis. Vaginal examination revealed that the uterus
5. Myelodysplastic was enlarged corresponding to 9 weeks of pregnancy due
to the fibromatous nodes, one of which was mobile and
184. An 8-month-old baby has decreased appetite, pale extremely painful. Appendages were not palpable. There
skin, enlarged right side of abdomen. Palpation the right were moderate mucous discharges. What is the optimal
side of abdomen reveals a dense elastic tumour-like treatment tactics?
formation 10×7 cm large. There is a positive ballotement 1. Surveillance and antibacterial therapy
sign. What is the most likely diagnosis? 2. Urgent surgery (laparotomy)
1. Colon pathology 3. Fractional diagnostic curettage of the uterine cavity
2. Congenital hydronephrosis 4. Surveillance and spasmolytic therapy
3. Liver tumour 5. Surgical laparoscopy
4. Intestinal tumour
5. Nephroblastoma (Wilms’ tumour) 189. A 19-year-old patient complains of dyspnea on
exertion. He often has bronchitis and pneumonia. Since
185. A 54-year-old patient complains of frequent painful childhood, the patient presents with cardiac murmur.
urination, chills, fever up to 38oC . Urine test results: Auscultation revealed splitting of the II sound above the
protein – 0,33 g/L, WBCs – up to 50-60 in the field of pulmonary artery, systolic murmur in 3 intercostal space at
vision, RBCs – 5-8 in the field of vision, gram-negative the left sternal border. ECG showed right bundle branch
bacilli. Which of the listed antibiotics should be preferred block. What is the provisional diagnosis?
in this case? 1. Open ductus arteriosus
Krok 2- 2013 (General Medicine) (t.me/krok2GeneralMedicine) - 24

2. Aortarctia 194. A 2-year-old child in a satisfactory condition


3. Aortic stenosis periodically presents with moderate proteinuria,
4. Atrial septal defect microhematuria. USI results: the left kidney is
5. Mitral insufficiency undetectable, the right one is enlarged, there are signs of
double pyelocaliceal system. What study is required to
190. A 55-year-old patient whose menstruation stopped 5 specify the diagnosis?
years ago complains of vaginal dryness, frequent and 1. Micturating cystography
painful urination. Gynecologist revealed signs of atrophic 2. Excretory urography
colpitis. Urine analysis revealed no peculiarities. Which 3. Radioisotope renal scan
locally acting product will provide the proper therapeutic 4. Retrograde urography
effect? 5. Doppler study of renal vessels
1. Vaginal cream "Meratin Combi"
2. Vaginal tablets "Tergynan" 195. A 13-year-old girl was admitted to the gynecological
3. Vaginal gel "Metronidazole" department with heavy bleeding, which appeared after a
4. Vaginal suppositories "Ovestin" long delay of menstruation. Shortly before, the girl
5. Vaginal cream "Dalacin" suffered a serious psychotrauma. Her menarche occurred
at the age of 11, she has a 30-day cycle with 5 to 6 days
191. A 22-year-old patient complains of amenorrhea for 8 of moderate, painless bleeding. The patient is somatically
months. Menarche occured at the age of 12,5. Since the healthy, of normosthenic constitution with height of 160
age of 18 the patient has a history of irregular cm, weight of 42 kg. The patient is pale. Rectoabdominal
menstruation. The patient is nulligravida. The mammary examination revealed that the uterus was of normal size
glands are developed properly, nipples discharge drops of and consistency, anteflexio-versio, the appendages were
milk when pressed. Gynecological study results: prolactin not changed. What is the most likely diagnosis?
level is 2 times higher than normal. CT reveals a bulky 1. Ovarian cyst
formation with a diameter of 4 mm in the region of sella. 2. Amenorrhea
What is the most likely diagnosis? 3. Hysteromyoma
1. Sheehan’s syndrome 4. Juvenile bleeding
2. Pituitary basophilia 5. Girl is healthy
3. Pituitary tumour
4. Lactational amenorrhea 196. In a cold weather, the emergency room admitted a
5. Stein-Leventhal syndrome patient pulled out of the open water. There was no
respiratory contact with the water. The patient is exci-ted,
192. After a contact with chemicals a plant worker has pale, complains of pain, numbness of hands and feet, cold
suddenly developed stridor, voice hoarseness, barking shiver. Breathing rate is 22/min, AP- 120/90 mm Hg, Ps-
cough, progressing dyspnea. Objective examination 110/min, rectal temperature is 34, 5oC . What kind of
reveals acrocyanosis. What is your provisional diagnosis? warming is indicated for this patient?
1. Laryngeal carcinoma 1. Hemodialysis with blood warming
2. PATE 2. Warm bath
3. Pneumothorax 3. Hot compresses
4. Pulmonary atelectasis 4. Passive warming
5. Laryngeal edema 5. Infusion of 37oC solutions

193. A 50-year-old patient has worked at a chemical plant 197. During self-examination a 22-year-old patient
for 15 years. His work involved using xylene solvent. The revealed a mammary tumour. Palpation revealed a firm,
patient was hospitalized with suspected chronic painless, mobile formation up to 2 cm, peripheral lymph
intoxication. He was found to have anemic syndrome. nodes were not changed. USI results: in the superior
What is the first-priority measure of secondary anemia external quadrant of the right mammary gland there was a
prevention? big formation of increased echogenicity, sized 18×17 mm.
1. Job change The patient was provisionally diagnosed with
2. Including meat into the diet fibroadenoma. What is a doctor’s further tactics?
3. Administration of iron supplements 1. Nonsteroid anti-inflammatory drugs, oral contraceptives
4. Administration of glucocorticosteroids 2. Surgical removal of the tumour prior to pregnancy
5. Including seafood into the diet 3. Dynamic follow-up
4. Surgical treatment after pregnancy
Krok 2- 2013 (General Medicine) (t.me/krok2GeneralMedicine) - 25

5. Radical mastectomy

198. A 28-year-old female patient has been admitted to a


hospital. She states to be ill for 12 years. On examination
she has been diagnosed with bronchiectasis with affection
of the left lower lobe of lung. What is the optimal treatment
tactics for this patient?
1. Active drainage of the left pleural cavity
2. Left-sided pneumoectomy
3. Antibiotic therpy
4. Bronchopulmonary lavage
5. Left lower lobectomy

199. Examination of a group of persons living on the same


territory revealed the following common symptoms: dark-
yellow pigmentation of the tooth enamel, diffuse
osteoporosis of bone apparatus, ossification of ligaments
and joints, functional disorders of the central nervous
system. This condition may be caused by the excessive
concentration of the following microelement in food or
drinking water:
1. Iodine
2. Fluorine
3. Copper
4. Cesium
5. Nickel

200. A 75-year-old male patient complains of slight pain in


the right iliac region. The abdominal pain arose 6 days
ago and was accompanied by nausea. Surgical
examination revealed moist tongue, Ps- 76 bpm. AP-
130/80 mm Hg. Abdomen was soft, slightly painful in the
right iliac region on deep palpation, the symptoms of the
peritoneum irritation were doubtful. In blood: RBCs – 4, 0 ·
12 9
10 /l, Hb- 135 g/l, WBCs – 9, 5 · 10 /l, stab neutrophils –
5%, segmentonuclear – 52%, lymphocytes – 38%,
monocytes – 5%, ESR – 20 mm/h. Specify the doctor’s
further tactics:
1. Emergency operation for acute appendicitis
2. Refer the patient to a district therapist
3. Hospitalization, dynamic surveillance
4. Send the patient home
5. Administration of additional examination: abdominal
ultrasound, x-ray contrast study of the gastrointestinal
tract
Krok 2- 2013 (General Medicine) (t.me/krok2GeneralMedicine) - 26

Krok 2- 2013 (General Medicine) Answer Key


01.(2) 31.(2) 61.(5) 91.(4) 121.(4) 151.(1) 181.(4)
02.(4) 32.(3) 62.(3) 92.(2) 122.(1) 152.(5) 182.(5)
03.(2) 33.(2) 63.(2) 93.(3) 123.(4) 153.(2) 183.(1)
04.(5) 34.(1) 64.(3) 94.(4) 124.(5) 154.(1) 184.(5)
05.(5) 35.(2) 65.(3) 95.(3) 125.(4) 155.(2) 185.(1)
06.(5) 36.(1) 66.(1) 96.(1) 126.(1) 156.(2) 186.(1)
07.(3) 37.(5) 67.(2) 97.(2) 127.(4) 157.(2) 187.(4)
08.(1) 38.(4) 68.(5) 98.(2) 128.(4) 158.(3) 188.(2)
09.(2) 39.(3) 69.(4) 99.(5) 129.(1) 159.(2) 189.(4)
10.(2) 40.(5) 70.(3) 100.(3) 130.(5) 160.(4) 190.(4)
11.(1) 41.(2) 71.(2) 101.(5) 131.(3) 161.(2) 191.(3)
12.(5) 42.(4) 72.(5) 102.(4) 132.(2) 162.(4) 192.(5)
13.(5) 43.(5) 73.(5) 103.(4) 133.(2) 163.(4) 193.(1)
14.(1) 44.(5) 74.(2) 104.(2) 134.(5) 164.(4) 194.(2)
15.(5) 45.(5) 75.(5) 105.(3) 135.(2) 165.(4) 195.(4)
16.(4) 46.(3) 76.(5) 106.(1) 136.(1) 166.(3) 196.(4)
17.(4) 47.(4) 77.(4) 107.(4) 137.(1) 167.(2) 197.(2)
18.(2) 48.(2) 78.(3) 108.(5) 138.(2) 168.(5) 198.(5)
19.(1) 49.(1) 79.(3) 109.(5) 139.(5) 169.(5) 199.(2)
20.(5) 50.(2) 80.(2) 110.(1) 140.(5) 170.(3) 200.(1)
21.(5) 51.(5) 81.(3) 111.(1) 141.(5) 171.(2)
22.(4) 52.(5) 82.(2) 112.(3) 142.(1) 172.(3)
23.(4) 53.(4) 83.(1) 113.(5) 143.(5) 173.(1)
24.(3) 54.(1) 84.(1) 114.(2) 144.(1) 174.(3)
25.(5) 55.(4) 85.(3) 115.(1) 145.(2) 175.(1)
26.(2) 56.(4) 86.(3) 116.(5) 146.(1) 176.(1)
27.(5) 57.(4) 87.(1) 117.(5) 147.(4) 177.(4)
28.(2) 58.(1) 88.(2) 118.(3) 148.(1) 178.(2)
29.(3) 59.(2) 89.(4) 119.(5) 149.(4) 179.(4)
30.(2) 60.(3) 90.(1) 120.(3) 150.(2) 180.(4)

You might also like